Download A 77-year-old white man comes to the clinic complaining of fatigue

Document related concepts

Dental emergency wikipedia , lookup

Prenatal testing wikipedia , lookup

Medical ethics wikipedia , lookup

Patient safety wikipedia , lookup

Electronic prescribing wikipedia , lookup

Dysprosody wikipedia , lookup

Adherence (medicine) wikipedia , lookup

List of medical mnemonics wikipedia , lookup

Transcript
A 77-year-old white man comes to the clinic complaining of fatigue and
shortness of breath. Physical examination reveals a palpable liver edge 3 cm
below the right costal margin and a 2+ pitting edema of the lower extremities.
You notice in his clinic chart that he initially presented 2 months earlier
complaining of a 4-month history of shortness of breath that was worse on
exertion and caused him to routinely awaken from sleep to go to the window
to "get air". You see that at that time in the chart it says that the "cardiac
examination is unremarkable and there is no jugular venous distension or
pitting edema. However, there are fine bibasilar crackles". You can see that
the patient was prescribed a medication, but unfortunately you cannot read
what it was and of course he did not bring the bottle with him. At this time, the
most likely underlying cause of this patient's condition is
A. asthma
B. biventricular heart failure
C. cor pulmonale
D. left heart failure
E. right heart failure
Explanation:
The correct answer is B. The patient has biventricular heart failure. After the
first visit, the patient was most likely treated with a diuretic. After the second
visit, the patient is now showing signs of biventricular heart failure. Signs of
left heart failure include dyspnea on exertion, which progressed to dyspnea
at rest, and paroxysmal nocturnal dyspnea in which the patient goes into
pulmonary edema when supine. In severe left heart failure, florid pulmonary
edema develops and is accompanied by cough and frothy sputum and
wheezing. When these symptoms, plus JVD, hepatojugular reflex,
hepatomegaly, and pitting edema are present, the right heart has failed also.
Asthma (choice A) is characterized by wheezing and shortness of breath. It
does not typically present with the physical signs (hepatomegaly, JVD) that
are present in this patient.
Cor pulmonale (choice C) is incorrect. Cor pulmonale is a condition in which
there is pulmonary hypertension leading to a failing right heart. The left
ventricle is not affected, and therefore this patient would not have crackles
and pulmonary edema.
Due to the presence of JVD and pitting edema, the patient does not solely
have left heart failure (choice D).
Due to the presence of crackles on pulmonary exam and shortness of breath
on exertion and paroxysmal nocturnal dyspnea, the patient does not solely
have right ventricular failure (choice E).
1
A 48-year-old man comes to the office complaining of "chest pain." He states
that he has been having episodes of "vague chest pain" for the past few
years when he is at home with his wife. The pain does not occur when he is
sleeping or when he is away from the house. He is a traveling salesman and
he says that he enjoys his work and likes "being on the road." His
temperature is 37 C (98.6 F), blood pressure is 120/80 mm Hg, and pulse is
60/min. Physical examination is unremarkable. An electrocardiogram is
normal. The most appropriate remark to this patient at this time is
A. "Do you feel the pain in your jaw when you feel it in your chest?"
B. "Does your wife know that you have chest pain?"
C. "Tell me how you feel when you have the pain."
D. "Why didn't you come to see me sooner?"
E. "You seem to have psychosomatic chest pain."
Explanation:
The correct answer is C. This patient's chest pain seems to be related to his
feelings towards his wife and his home, and it is therefore best to start with
an open-ended statement such as "Tell me how you feel when you have the
pain?" This allows him to talk freely and can generally lead to a lot of
information. By asking how he "feels" when he has the pain, you may be
able to elicit the psychological feelings that are likely related to his chest
pain.
"Do you feel the pain in your jaw when you feel it in your chest?"(choice A) is
not the best choice, because this is a suggestive, direct question. An openended question typically elicits more information and allows the patient to
lead the discussion in a direction that they think is relevant. However, it may
lead to disastrous rambling that requires a more direct line of questioning. It
is suggestive, because it basically gives the patient the answer you are
looking for. If you want to know if the pain radiates, you should ask, “Do you
feel the pain anywhere else when you feel it in your chest?” This open
ended question should elicit a more accurate answer.
"Does your wife know that you have chest pain?" (choice B), is not a good
question at this time, because it will not provide the most relevant
information that is necessary at this time. You need to know how he feels
about his chest pain, and what exactly is going on with him that is causing
the pain. Later on, if he says that it is related to his feelings towards her, you
may want to ask if she is aware of his feelings, but it is not the most
appropriate question at this time.
"Why didn't you come to see me sooner?" (choice D), is a bad question
because it sounds accusatory and will make him defensive.
"You seem to have psychosomatic chest pain." (choice E), is inappropriate,
because it is medical jargon, and it is incorrect to jump to conclusions
without allowing him to freely discuss his pain and feelings about the pain.
2
An open-ended question or statement will generally provide the most
information, and should therefore, be used first. Later on in the conversation
it may be appropriate to say, "your chest pain appears to be related to your
feelings towards your wife and your home."
A 59-year-old man is admitted to the hospital for shortness of breath. The
patient has a long-standing cardiac history and has suffered two non-Q wave
infarctions in the past 20 months. The patient reports bright red blood in the
toilet bowl during his last bowel movement. Laboratory data are remarkable
for a hematocrit of 22%. Given the patient's known coronary disease, his
attending cardiologist recommends a blood transfusion. As appropriate, you
order 2 units of appropriately matched red bloods cells in order to transfuse
the patient to a target hematocrit above 30%. While the first unit is being
administered, the patient becomes febrile and develops chest and flank pain.
You are immediately summoned to his side and on arrival you note erythema
around the intravenous access site and a small volume of dark colored urine
in his Foley catheter bag. The remainder of the physical examination is
unremarkable. The most likely diagnosis is
A. acute febrile hemolytic reaction
B. anaphylaxis
C. delayed hemolytic transfusion reaction
D. pulmonary embolus
E. transfusion associated lung injury
Explanation:
The correct answer is A. This patient is most likely having an acute febrile
hemolytic reaction. This occurs when there is a mismatch between the
patient's and donor's ABO types and classically presents with fever,
chest/back pain, nausea, pain, and erythema around the infusion site and
hemoglobinuria.
Anaphylaxis (choice B) is unlikely given that the patient has no known drug
allergies. Also, there is no mention of the bronchospasm or urticaria that
would be expected with this process.
A delayed hemolytic transfusion reaction (choice C) is also unlikely given the
acuity of presentation. This reaction typically presents about 3-10 days
following a transfusion reaction and has a mild course with fever and
malaise.
A pulmonary embolus (choice D) becomes less likely given that there is no
mention of dyspnea or impaired oxygenation. There also is no mention of
the classic physical stigmata of a pulmonary embolus such as a loud
pulmonic component of the second heart sound or pulmonic tap.
Transfusion associated lung injury (choice E) is unlikely given the acuity of
3
symptoms. This process, which typically manifests as an acute lung injury
type pathology, typically presents with a more delayed course.
A 68-year-old male is brought to the emergency department because of substernal chest
pain. On interview, the patient reports having a past myocardial infarction 5 years ago.
He has typical anginal pain, when climbing stairs and when lifting heavy objects, that is
relieved by taking nitroglycerin tablets and resting. In the last week, however, his anginal
symptoms have become more frequent and occur when walking even short distances. In
the hospital, the patient continues having chest pain over his left side accompanied by
shooting pain in his left arm. His blood pressure is160/90 mm Hg, pulse is 109/min, and
oxygen saturation is 96%. Physical examination is unremarkable. A chest x-ray is clear
with normal cardiac size. An electrocardiogram shows Q-waves in leads II, III, and AvF
and T-wave inversions in v1-v3. Laboratory studies show a potassium of 3.2 mEq/L,
hematocrit of 42%, and initial cardiac markers are negative. The patient receives
oxygen, an aspirin, a beta-blocker, intravenous heparin, and is placed on a platelet gp
IIb/IIIa inhibitor. In addition, he is made pain-free on intravenous nitroglycerin. A repeat
electrocardiogram without pain is unchanged. The most appropriate next step in
management is
A. admission to the hospital and observation for 48 hours
B. coronary angiography
C. emergent coronary artery bypass graft (CABG)
D. intra-aortic balloon pump
E. thrombolysis by administration with tPA
Explanation:
The correct answer is B. The patient's presentation is highly worrisome for an acute
coronary syndrome and therefore requires a coronary angiogram. From his past
medical history, the patient has known coronary artery disease. His recent history is a
classic description of crescendo angina. On presentation, he complains of chest pain at
rest and negative cardiac markers indicating likely unstable angina. His vital signs are
stable and there is no evidence of acute congestive heart failure or pulmonary edema
by CXR and oxygen saturation. The EKG indicated an old inferior myocardial infarction,
but more importantly, the EKG shows T-wave inversions in the anteroseptal precordial
leads. This may suggest critical left main coronary artery disease or stenosis (Wallen
sign). The patient needs to undergo coronary angiography for several reasons. First,
data has shown improved outcomes in patients with unstable angina and primary
coronary intervention, so if there is a culprit lesion that can be fixed, (i.e., by stenting), it
should be. Secondly, the anterior wall of the left ventricle is at risk. The patient has
already had damage to his right ventricle (inferior Q waves), and now his left wall is
threatened. Third, the EKG indicates Wallen sign: T-wave inversions in v1-v3 or v4.
This correlates to possible left main coronary artery (LMCA) disease. LMCA stenosis
risks both the anterior wall and the lateral wall of the left ventricle. Due to the large area
at risk, the patient must undergo CABG , and not PTCA. However, without coronary
angiography, it is unknown whether the patient has LMCA disease or a proximal LAD
4
lesion.
It is inappropriate to admit him for observation only (choice A). Although the patient has
been placed on adequate medications and may potentially improve on medical
management, several studies have shown improved outcomes in patients taken to
catheterization with acute coronary syndromes. In addition the EKG suggests that the
anterior wall may be at risk and this would have serious consequences if allowed to
proceed to infarction. The goal is to re-establish perfusion to that area as soon as
possible.
The patient is stable and it is not known for certain whether or not he requires a CABG
(choice C) for LMCA. If he simply has a proximal LAD lesion, he can undergo PCI
without a need for cardiac surgery.
His vital signs are stable and he is not in cardiogenic shock. In addition, he has no
history of valvular disease. Therefore, there is no indication for an intra-aortic balloon
pump (choice D).
Criteria for thrombolytics (choice E) requires ST segment elevations >1 mm in 2
contiguous leads, ST segment depressions >2mm in the anterior leads (v1-v2), or new
left-bundle branch block. The patient does not meet these criteria.
A 66-year-old patient who is well known to you presents to your office three
weeks after being discharged from the hospital after suffering a myocardial
infarction. The patient suffered an acute anterior wall Q-wave infarct and was
treated by thrombolysis at the local hospital. The patient did well on post-lytic
therapy and was discharged six days after the event. The patient is very
concerned that this is his second heart attack. You decide to spend additional
office time with him ensuring that all of his questions are answered and that
his medical regimen is optimal. In reviewing his medication list, there are a
number of medications that the patient could be on that he currently is not.
His current regimen includes atenolol, simvastatin, and nifedipine. The most
appropriate new medication to add to his regimen is
A. aspirin
B. digoxin
C. enalapril
D. propranolol
E. warfarin
Explanation:
The correct answer is A. Aspirin at a dose of 81mg to 325mg per day should
be continued indefinitely in a post MI patient. Although individual trials have
been unconvincing in demonstrating a benefit for aspirin for secondary MI
prevention, at least two meta-analyses have clearly shown a significant
benefit to taking aspirin post MI.
5
Digoxin (choice B) is not specifically indicated in post MI populations, but it is
routinely used as therapy for congestive heart failure. It has no mortality
benefit when used in such patients, although it does improve congestive
symptoms.
ACE inhibitors (choice C) have also been shown to be beneficial in the post
MI setting, especially in patients with poor LV function. However, the
magnitude of this benefit, like that of beta blockers, is not more impressive
that that of aspirin.
Beta blockers (choice D) have been shown in a number of trials to improve
mortality when given in the post MI period. Atenolol is a long-acting, once
daily beta blocker. In fact, recent studies even suggest that these agents are
underutilized by physicians. However, use of such agents in the post MI
period is not without risk, and unless an ejection fraction is known and if any
heart block or bradycardia is present, these agents should not be used
indiscriminately. He is already taking atenolol; therefore adding propranolol
is not indicated.
Warfarin (choice E) has been shown to be beneficial in patients not receiving
anti-platelet therapy. It appears that the routine use of aspirin can account
for all of the beneficial effects of warfarin or survival in post MI patients.
A mother brings her 5-year-old boy to the clinic because of a rash on his legs
and buttocks that she noticed this morning. He has also been complaining
that his "belly hurts," but has had no change in appetite. He had an upper
respiratory tract infection and sore throat about 1 week ago. He has not had
any fevers, recent weight loss or joint pain, and has not taken any
medications. His temperature is 37.0 C (98.6 F). Physical examination shows
mild periumbilical tenderness and multiple 3-6 mm raised erythematous
lesions on his lower extremities and buttocks. The lesions do not blanch with
pressure. His leukocyte count, hemoglobin, platelet count, and coagulation
studies are normal. Urinalysis shows 3-5 RBCs per hpf. A rapid strep test is
positive. The most likely diagnosis is
A. Henoch-Schönlein purpura
B. Idiopathic thrombocytopenic purpura
C. Kawasaki disease
D. Rocky Mountain spotted fever
E. Wiskott-Aldrich syndrome
Explanation:
The correct answer is A. This patient presents with a classic case of
Henoch-Schönlein purpura (HSP). HSP is an Ig-A mediated vasculitis
involving the small blood vessels (arterioles and venules) of the skin, GI
tract, kidneys, and joints. It is the most common vasculitis affecting children.
6
The mean age is 4-7 years of age with slight male predominance. About
50% of children have a preceding upper respiratory tract infection and about
75% have group A strep recovered from their oropharynx. The classic rash
is described as palpable purpura concentrated on the buttocks and lower
extremities, but rarely on the trunk. These children can also present with
non-migratory arthritis, colicky abdominal pain, microscopic or gross
hematuria indicative of nephritis, and males can present with scrotal
swelling. These patients must have a normal or elevated platelet count. This
is what differentiates HSP from idiopathic thrombocytopenic purpura (ITP)
(choice B). ITP also presents with a purpuric rash, but these children have
low platelet counts. It is thought that these children develop an autoantibody
to the platelet surface after a viral illness.
Kawasaki disease (choice C) is also known as mucocutaneous lymph node
syndrome. It is an acute febrile vasculitis, which primarily affects the
medium-sized vessels with particular affinity to the coronary arteries.
Children must meet certain diagnostic criteria to be given the diagnosis of
Kawasaki disease. They include fever for at least 5 days, plus four of the
following five criteria: bilateral conjunctival injection; injected pharynx/dry or
fissured lips/strawberry tongue; edema/erythema of the hands and feet; nonvesicular truncal rash; and cervical lymphadenopathy. The patient in the
vignette does not meet the criteria for Kawasaki disease.
Rocky Mountain spotted fever (choice D) is an infectious disease caused by
the bacterium Rickettsia rickettsii. These patients will present with fever,
headache, malaise, and the classic rash is maculopapular and begins on the
extremities and spreads inward to the trunk and includes the palms and
soles. The rash may become purpuric after several days.
Wiskott-Aldrich syndrome (choice E) is an X-linked recessive syndrome
characterized by eczema, thrombocytopenia, petechiae, and recurrent
infections. This presentation is quite different from the boy in this vignette.
A 73-year-old woman comes to the office because of "crampy" leg pain
experienced over the past 6 months, while walking her dog. The pain is only
present during exertion and is relieved by rest. She has had stable angina for
the past 20 years. She smokes 1 1/2 packs of cigarettes a day. She does not
take any medications. Her blood pressure is 130/90 mm Hg and pulse is
75/min. Physical examination shows a diminished femoral pulse on the right
leg compared to the left and absent popliteal and pedal pulses on the right
leg. There is no hair below the knee on the right leg, the skin is shiny and
smooth, and the toenails are thickened. Neurological examination is
unremarkable. The ankle to brachial artery pressure ratio is 0.7. A Doppler
device shows decreased blood flow to the right leg. You should
A. advise her to quit smoking and begin a progressively strenuous
exercise program
B. recommend elastic support hose
7
C. refer her to a vascular surgeon for evaluation for bypass surgery
D. schedule angiography of the right leg
E. schedule percutaneous transluminal angioplasty
Explanation:
The correct answer is A. This patient is describing intermittent claudication,
which is a sign of peripheral vascular disease. Patients often complain of
pain during exertion that is relieved by rest. The physical findings in this
case are very consistent with this diagnosis. Noninvasive evaluation is
usually recommended initially and consists of determining the ratio of ankle
to brachial arterial pressures. The ankle/brachial artery ratio is determined
by measuring and comparing the 2 blood pressures. If the ratio is less than 1
(greater than or equal to 1 is considered normal), peripheral artery disease
is present. A ratio less than 0.5 is considered severe ischemia. Since this
patient's ankle to brachial artery pressure ratio is 0.7 and you note that there
is decreased blood flow to the right leg, therapeutic options should be
discussed. The best initial treatment for this patient at this time is to quit
smoking and begin a progressively strenuous exercise program. You should
also advise her to maintain meticulous foot care and begin therapy with
aspirin or another antiplatelet drug.
Elastic support hose (choice B) should not be worn by patients with
peripheral vascular disease and claudication because it reduces blood flow
to the skin. Support hose are recommended for patients with varicose veins.
It is inappropriate to refer her to a surgeon for an evaluation for bypass
surgery (choice C) at this time. This patient should be advised to quit
smoking, begin a progressively strenuous exercise program, maintain
meticulous foot care, and possibly begin therapy with aspirin or another
antiplatelet drug. Surgery is usually only indicated for severe, disabling
disease.
It is inappropriate to schedule angiography of the left leg (choice D) at this
time. Angiography is not used as a routine diagnostic study for the initial
evaluation of claudication. It should only be performed if surgical
revascularization is being considered. Supportive and nonoperative
measures are indicated in this case.
Percutaneous transluminal angioplasty (choice E) is a procedure that is
usually only indicated for severe, disabling disease. This catheter-based
intervention is considered a nonoperative revascularization procedure, but it
is still usually reserved for severe disease.
You are called to see a 72-year-old man with metastatic pancreatic
adenocarcinoma who was admitted to the hospital for a palliative
gastrojejunostomy. His hospital course has been unremarkable but today he
developed hematemesis. His current medications include multivitamins,
folate, and a fentanyl patch. When you arrive at his room you see a thin,
8
jaundiced man with a nasogastric tube in place and multiple petechiae and
dried blood around his nares and mouth. His temperature is 37.0 C (98.6 F),
blood pressure is 98/67 mm Hg, pulse is 103/min, and respirations are
25/min. He has faint crackles bilaterally on auscultation. The remainder of the
examination is unremarkable. His platelet count is down to 45,000/mm3 from
120,000/mm3 3 days earlier. His primary oncologist is concerned that this
patient has developed disseminated intravascular coagulopathy. The most
appropriate next step in evaluation is to
A. determine erythrocyte sedimentation rate
B. determine prothrombin and partial thromboplastin time
C. determine thrombin levels
D. send stool study for Escherichia Coli O157:H7
E. send urine analysis for bence jones protein
Explanation:
The correct answer is B. Disseminated intravascular coagulopathy (DIC) is
characterized by a state where the thrombotic and fibrinolytic processes of
the body are both activated leading to either bleeding (most often) or
thromboses. Clotting factors are depleted given these ongoing thromboses.
DIC is, therefore, characterized by thrombocytopenia (from platelet
consumption) with an elevated prothrombin and partial thromboplastin time.
An erythrocyte sedimentation rate (choice A) is a non-specific indicator of an
inflammatory process and has no role in the diagnosis of disseminated
intravascular coagulopathy.
A thrombin level (choice C) is not useful in the diagnosis of disseminated
intravascular coagulopathy.
A stool study for Escherichia Coli O157:H7 (choice D) is used for the
diagnosis of hemolytic uremic syndrome, syndrome characterized by
thrombocytopenia with normal prothrombin and partial thromboplastin times.
It is not associated with a bleeding diathesis.
A urine analysis for bence jones proteins (choice E), is useful in the
diagnosis of disorders with elevated serum proteins such as with multiple
myeloma. It has no role in the diagnosis of disseminated intravascular
coagulopathy.
A 55-year-old woman is brought to your emergency department complaining
of severe substernal chest “pressure”. Her medical history includes a history
of coronary artery disease with 2 previous myocardial infarctions,
hypertension, hyperlipidemia, and diabetes. Her blood pressure is 108/65 mm
Hg, pulse is 100/min and regular, and respirations are 22/min. Physical
examination shows warm, moist skin and clear breath sounds bilaterally.
Cardiac examination is unremarkable. An electrocardiogram shows sinus
9
rhythm with ST elevations over the anterior leads. After administering aspirin,
the most appropriate intervention is to
A. admit her for emergent cardiac catheterization
B. admit her for emergent coronary artery bypass grafting
C. admit her for insertion of an intraaortic balloon pump (IABP)
D. immediately attempt cardioversion
E. immediately defibrillate her
Explanation:
The correct answer is A. The standard of care for any patient who presents
with evidence of an ST elevation myocardial infarction is an emergent
cardiac catheterization. If one or more discrete lesions are found during a
catheterization, they can be angioplastied or stented, if amenable.
Emergent coronary artery bypass grafting (choice B) is reserved until
cardiac catheterization and a definition of the diseased coronary anatomy is
available.
Insertion of an intraaortic balloon pump (IABP) (choice C) is reserved for
patients who have severe ventricular dysfunction or refractory ischemia.
Given this patient's hemodynamics, and since no interventions have yet
been attempted, it is not merited at this juncture.
Cardioversion (choice D) is reserved for stable dysrhythmias such as stable
atrial fibrillation or atrial flutter. It, like defibrillation, has no role in the
management of this patient who is in sinus rhythm.
Defibrillation (choice E) is used for the treatment of ventricular fibrillation,
unstable atrial fibrillation, or rapid ventricular tachycardias. It has no role in
the management of this patient who is in sinus rhythm.
A 40-year-old man comes to the office complaining of a 3-day history of
midsternal chest pain, non-radiating that is worse with inspiration and relieved
by sitting forward. He has no past medical history, is on no medications, does
not smoke, and has no known drug allergies. He leads an active lifestyle, and
had been running about 10 miles a week without problem until a week ago
when he developed a “viral syndrome.” His temperature is 38.4 C (100 F),
blood pressure is 130/70 mm Hg, pulse is 100/min and regular, and
respiratory rate is 20/min. He has a high pitched, grating sound that can be
auscultated throughout the cardiac cycle over his precordium. An
electrocardiogram shows diffuse ST elevation, diffuse PR depression with PR
elevation in lead aVR. The most likely diagnosis is
A. angina
10
B. myocardial infarction
C. pericarditis
D. pneumonia
E. pulmonary embolism
Explanation:
The correct is answer is C. A chest pain that is pleuritic and improves with
sitting up and leaning forward is a classic description of the chest pain
associated with pericarditis, as is the precordial rub auscultated on physical
exam. Diffuse ST elevations with diffuse PR depressions and PR elevation
in lead aVR is also the classic description of the EKG findings associated
with pericarditis.
Angina (choice A) or myocardial infarction (choice B) are less likely given
that the patient has no risk for coronary artery disease (hypertension,
hyperlipidemia, diabetes, tobacco). He also describes a very active lifestyle
without symptoms, suggestive of no cardiac pathology prior to the onset of
these symptoms. The ST changes on EKG, that alone would suggest
ischemia, become more consistent with pericarditis since they are
associated with PR segment changes that are classically associated with
this particular disease process.
Pneumonia (choice D) is unlikely since there is no history of cough or
dyspnea, and there is no mention of findings on the chest exam consistent
with such a process (e.g., findings suggesting a consolidative process such
as decreased breath sounds, increased fremitus, and egophony over the
affected area).
Pulmonary embolism (choice E) is less likely, given that there are no
apparent risk factors such as a history of hypercoagulability or poor activity.
The EKG findings are also not consistent with a pulmonary embolism, where
one would classically see an S wave in lead I, Q wave and T wave inversion
in lead III, or ST-T wave changes in leads V1 through V4.
A 49-year-old man with AIDS comes to the clinic with unexplained shortness
of breath for the past month. He is otherwise asymptomatic and his
medications include AZT, indinavir, and trimethoprim/sulfamethoxazole. His
temperature is 37.0 C (98.6 F), blood pressure is 110/70 mm Hg, and
respirations are16/min. Physical examination reveals diminished heart
sounds, but is otherwise unremarkable. An electrocardiogram reveals normal
sinus rhythm at a rate of 90/min and low voltages in all leads. Posteroanterior
and lateral chest x-rays demonstrate minimally increased interstitial markings,
an enlarged cardiac silhouette, and no focal consolidation or pleural
effusions. A CT scan of the chest with intravenous contrast is shown.
11
This patient's shortness of breath is most likely due to
A. bacterial endocarditis
B. bacterial pneumonia
C. a myocardial infarction
D. a pericardial effusion
E. a pneumothorax
Explanation:
The correct answer is D. The shortness of breath, diminished heart sounds,
low voltage on electrocardiogram, enlarged heart on x-rays and chest CT
(fluid attenuation in the pericardium without pericardial thickening) all
support a pericardial effusion. Pericardial effusions may be idiopathic or
infectious in patients with AIDS.
Bacterial endocarditis (choice A) would present with a fulminant course of
high fever, septic emboli, sepsis, and possibly heart failure.
Bacterial pneumonia (choice B) presents with fever, sepsis, and focal lung
consolidation.
A myocardial infarction (choice C) would be unusual in a 49-year-old without
severe coronary artery disease. AIDS itself is not a predisposing factor. The
12
electrocardiogram did not demonstrate any evidence of ischemia or infarct.
A pneumothorax (choice E) may present spontaneously. There is no
pneumothorax on the chest x-rays or the chest CT.
A 79-year-old woman with a history of polymyalgia rheumatica and
osteoarthritis comes to the office complaining of periodic severe left sided jaw
pain, headaches, and blurry vision. Her temperature is 38.1 C (100.5 F),
blood pressure is 150/70 mm Hg, pulse is 73/min, and respirations are
13/min. Physical examination is significant for left sided scalp tenderness.
Laboratory studies show:
The most appropriate next step in the management of this patient is to begin
therapy with
A. atenolol
B. diazepam
C. ibuprofen
D. isosorbide mononitrate
E. prednisone
Explanation:
The correct answer is E. This patient likely has temporal arteritis, which is a
vasculitis predominantly of the elderly associated with polymyalgia
rheumatica. An elevated ESR is typically seen in this condition. Diagnosis is
confirmed by superficial temporal artery biopsy demonstrating
granulomatous inflammation. Features of the disease include headaches,
visual symptoms, jaw claudication, scalp tenderness, and varying
neurological complaints. These symptoms are secondary to ischemia of the
temporal artery. The treatment is high-dose steroids ASAP to prevent
blindness. Missing this diagnosis is easy but its effects can be disastrous.
Atenolol (choice A) is a reasonable choice to manage this patient's
hypertension, but it is unlikely that her mild elevation in blood pressure is
leading to visual changes and headaches. You would expect her blood
pressure to be significantly higher to realistically attribute her complaints to
13
those levels.
Diazepam (choice B) is an excellent muscle relaxant and can be useful for
severe muscle spasm. Muscle spasm is an unlikely diagnosis in this patient.
Ibuprofen (choice C) is not appropriate for this patient's presentation.
Nitrates (choice D) would be reasonable if you suspected that this patient's
jaw pain was an anginal equivalent since the elderly and women can have
atypical angina, but steroids are the better answer in this patient. If a biopsy
reveals no evidence of temporal arteritis, nitrates along with a stress test
would be indicated.
A 36-year-old man is admitted to the hospital for severe hypertension. He has
had high blood pressure for the past 3 years that has been very difficult to
control. There is no history of hypertension in his family and he has no other
medical problems. His current medications include hydralazine, amlodipine,
and atenolol. His blood pressure log-book that he keeps at home shows that
his daily pressures have been on average 180/90 mm Hg. Today he was
admitted for a blood pressure of 220/120 mm Hg with pulse of 82/min. On
physical examination, he is appropriately anxious but in no distress. He fundi
are clear with no evidence of papilledema. His heart exam is benign. An
electrocardiogram shows left ventricular hypertrophy at 80 beats per minute
with no strain pattern. Laboratory studies show:
Sodium
151 mEq/L
Potassium 2.6 mEq/L
Bicarbonate 28 mEq/L
BUN
10 mg/dL
Creatinine 0.8 mg/dL
The most appropriate diagnostic test at this time is
A. an abdominal CT scan
B. a head CT scan
C. a renal scan
D. a renal vein renin level
E. urine catecholamine and VMA levels
Explanation:
The correct answer is A. It is generally true that greater than 90% of patients
with hypertension have essential hypertension. The remainder have a
variety of secondary causes, all of which are in general rare. For this patient,
hypernatremia and hypokalemia imply excess aldosterone. The electrolyte
imbalance coupled with the hypertension strongly suggests Conn's disease
or primary hyperaldosteronism. The imaging modality of choice would be the
14
one that looks for an adrenal mass.
A head CT scan (choice B) is not indicated as there is no evidence for any
intracranial pathology.
A renal scan (choice C) is used to assess for renal artery stenosis and
possible renovascular hypertension. In this case, the electrolyte disturbance
is so characteristic of elevated aldosterone, that RAS is not a diagnosis to
pursue.
Renal vein renin levels (choice D) are used to assess for possible secondary
aldosteronism. This is a useful diagnostic test but cumbersome to perform
and should not be done prior to the potentially high yield and simple
abdominal CT scan.
Urine catecholamine and VMA levels (choice E) are useful in the diagnosis
of pheochromocytoma. This is also a rare cause of hypertension but is less
likely than aldosteronism because excess catechols are not associated with
salt retention and potassium wasting.
A 35-year-old man comes to the emergency department with a 2-day history
of sharp chest pain that has been getting progressively worse. The pain
increases when he is supine and improves when he sits upright. He has
never had pain like this before and he denies dyspnea, diaphoresis, nausea,
or presyncope. He does not smoke, has no chronic medical conditions, and
has no family history of coronary artery disease. He is an avid jogger and was
running 3-5 miles every day up until the pain began 2 days ago. Upon further
questioning, you discover that he had a recent upper respiratory tract
infection that resolved approximately 5 days ago. His temperature is 37.6 C
(99.7 F), blood pressure is 120/82 mm Hg, pulse is 95/min, and respirations
are 14/min. Physical examination shows a triphasic cardiac rub. A chest x-ray
is normal and an electrocardiogram shows diffuse ST segment elevation with
ST segment depression in aVR. The most appropriate next step in
management is to
A. administer nonsteroidal antiinflammatory agents
B. obtain an echocardiogram
C. obtain serial cardiac enzymes
D. order serologies for coxsackie B virus
E. treat the patient with a second-generation cephalosporin
Explanation:
The correct answer is A. The patient has acute pericarditis, which is purely a
clinical diagnosis. There is no single serologic or imaging test that confirms
the diagnosis. The classic pleuritic chest pain that worsens when supine, the
pericardial friction rub on exam, and the diffuse ST segment elevation on
ECG together make the diagnosis. Acute pericarditis may occur around the
15
time of a known viral or upper respiratory tract infection and may itself be
accompanied by fever. If the diagnosis remains uncertain, serologic markers
of inflammation (such as an elevated white count, erythrocyte sedimentation
rate, or C-reactive protein) may help make the diagnosis. The pericardial rub
is classically "triphasic". The three components come from atrial systole,
ventricular systole, and ventricular relaxation during ventricular diastole. The
treatment is nonsteroidal antiinflammatory drugs (NSAIDs). Naproxen and
indomethacin are most often used, but there is no data to demonstrate that
any one NSAID is superior to another. There is also little data on duration of
therapy, but a 7- to 14-day course is reasonable. The patient should follow
up with a physician during that time frame to assure that the pericarditis has
resolved.
An echocardiogram (choice B) is incorrect because there is no evidence of
congestive heart failure or hemodynamic compromise. Echocardiogram is
neither sensitive nor specific for the diagnosis of acute pericarditis and
should therefore not be used to rule in or rule out the disease. While
pericarditis can sometimes lead to a significant pericardial effusion and
rarely to pericardial tamponade, the patient had no evidence of pericardial
tamponade on history, no shortness of breath on exam, no hypotension,
distended neck veins, or pulsus paradoxus.
While myocardial infarction should always be on the differential diagnosis for
any patient presenting with chest pain, obtaining serial cardiac enzymes
(choice C) is incorrect for several reasons. First, he has no risk factors for
coronary artery disease. He is young, has no known hypertension or high
cholesterol, does not smoke, nor does he have a family history of coronary
artery disease. He jogs regularly without difficulty. The quality of his chest
pain (sharp and changing with position) would be atypical for ischemic pain.
The diffuse ST segment elevation on electrocardiogram is classic for
pericarditis and would have to represent the unlikely, simultaneous
involvement of several coronary arteries to reflect an acute myocardial
infarction.
In the majority of cases, acute pericarditis is idiopathic. Obtaining serologies
for coxsackie B virus (choice D) is incorrect because, while coxsackie B is
the most common known infectious etiology of acute pericarditis, the
presence or absence of the disease does not change management.
Treating the patient with a second-generation cephalosporin (choice E) is
incorrect because acute pericarditis is rarely of bacterial etiology. In cases
where the etiology is bacterial, patients will typically present quite ill, with
fever and other systemic signs or symptoms pointing to a specific disease
process.
An 84-year-old man is brought to the emergency department with
hemoptysis. His medical history is significant for coronary disease,
symptomatic carotid artery stenosis which is uncorrected, and hypertension.
His medications include warfarin, aspirin, metoprolol, and furosemide. The
patient appears anxious. His temperature is 37 C (98.6 F), blood pressure is
16
97/56 mm Hg, pulse is 119/min, and respirations are 25/min. Breath sounds
are absent over his left lower lung field. A chest radiograph shows
opacification of his left lower lung. Laboratory studies show a hematocrit of
32% and prothrombin time of 28 seconds. His blood type is AB. The most
appropriate management is
A. administration of type A fresh frozen plasma
B. administration of type B fresh frozen plasma
C. administration of type AB fresh frozen plasma
D. administration of type O fresh frozen plasma
E. administration of vitamin K
Explanation:
The correct answer is C. Plasma will contain antibodies against ABO
antigens not present on donor red cells. Therefore, a recipient whose blood
type is AB can receive plasma only from a donor whose blood type is also
AB. In an emergency, type O packed cells can be given, which should avoid
the anti-A and anti-B antibodies in the donor's plasma.
Administration of type A fresh frozen plasma (choice A) will lead to ABO
incompatibility and immediate hemolysis since type A plasma will contain
antibodies against B antigens present on the patient's red cells.
Administration of type B fresh frozen plasma (choice B) will lead to ABO
incompatibility and immediate hemolysis since type B plasma will contain
antibodies against A antigens present on the patient's red cells.
Administration of type O fresh frozen plasma (choice D) will lead to ABO
incompatibility and immediate hemolysis since type O plasma will contain
antibodies against A and B antigens present on the patient's red cells.
Since the beneficial effect of vitamin K on a warfarin-induced coagulopathy
does not manifest acutely, the administration of vitamin K (choice E) is
inappropriate in this setting due to the ongoing hemorrhage.
A 19-year-old woman comes to the student health service complaining that
since the new semester has begun, she finds herself unable to focus and
concentrate as well as before. She attributes this largely to feeling fatigued.
She denies any other symptoms such as sadness, sleeplessness, or loss of
libido. She has no other medical issues. Her medications are only oral
contraceptive pills. She has never been pregnant and denies current
pregnancy. She has a history of long menses, often lasting 8 days.
Laboratory studies show:
17
The most appropriate next step is to
A. administer iron, intravenously
B. administer vitamin B12, intravenously
C. advise her to take folate tablets
D. change her oral contraceptive formulation to estrogen only
E. tell her to take oral iron tablets
Explanation:
The correct answer is E. This patient has iron-deficiency anemia, a very
common diagnosis in women of childbearing age. This is a type of microcytic
anemia characterized by MCV <80 and a ferritin level less than 12 mcg/L.
The loss of iron for both men and women is about 30gm per month. For
menstruating women, it can be twice this. For this reason, iron deficiency is
prevalent in this population. Oral iron tables are the therapy of choice. They
are usually well tolerated and often remedy the problem within 12 weeks.
Intravenous iron (choice A) is not routinely used in the management of irondeficiency anemia. It does have some indications, for example in
hospitalized patients unable to take oral medications or solids due
maldigestion or malabsorption.
Vitamin B12 (choice B) is used to treat macrocytic anemia (MCV >100).
Folate (choice C) is used for macrocytic anemia as well and in pregnant
women for the prevention of neural tube defects. Serum B12 and folate
levels can be checked once the diagnosis of macrocytic anemia is made, but
often, both are prescribed since persons deficient in one are often, for
dietary reasons, deficient in the other.
Changing her oral contraceptive formulation to estrogen only (choice D) may
help her to remedy her excessive menses, but will do nothing to increase
her iron stores and correct her anemia. Therapy in this case is first directed
towards fixing the anemia.
You have been following a 12-year-old girl who was diagnosed with
autoimmune thrombocytopenic purpura of childhood (childhood ITP) 1 year
ago following a viral illness. She has continued to have thrombocytopenia
despite medical therapy. She recently received prednisone for 2 weeks
followed by 2 days of intravenous immune globulin therapy. Her platelet count
recently dropped below 20,000/mm3 requiring platelet transfusion and she
repeatedly presents with diffuse petechiae and epistaxis. You and your
colleagues decide that a splenectomy is the next step in treatment due to her
18
persistent and dangerously low platelet count. Following the splenectomy and
an uncomplicated postoperative course, she returns to your clinic for follow
up. The thrombocytopenia has resolved and she has clinically improved. The
most appropriate next step in this patient's management includes
A. a 1-month course of penicillin prophylaxis and influenza vaccine
B. a 1-month course of prednisone with concomitant IVIG therapy
C. permanent penicillin prophylaxis, pneumococcus vaccine, and H.
influenza vaccine
D. permanent prednisone therapy and influenza vaccine
E. a 2–week course of prednisone therapy
Explanation:
The correct answer is C. Autoimmune thrombocytopenic purpura of
childhood (childhood ITP) is a disorder that usually occurs after a viral
illness. The pathophysiology involves antibody (IgG or IgM) binding to
platelets. These antibody- coated platelets are subsequently destroyed in
the spleen. Thrombocytopenia ensues and most often resolves
spontaneously within 6 months. Persistent thrombocytopenia is treated with
2 weeks of prednisone 2-4 mg/kg/day or IVIG 1g/kg/day. Cases refractory to
medical therapy in which severe thrombocytopenia persists are treated with
splenectomy to prevent further destruction of platelets. An extremely
important aspect of management of the asplenic patient includes permanent
penicillin prophylaxis in addition to pneumococcal and Haemophilus
influenza vaccines. These measures decrease the risk of morbidity and
mortality associated with overwhelming sepsis by encapsulated organisms
in asplenic patients.
One month of penicillin prophylaxis and influenza vaccine (choice A) is an
inappropriate choice. Patients should remain on penicillin prophylaxis due to
the continued risk of infection. Asplenic patients are susceptible to infection
with encapsulated organisms including Pneumococcus and H. influenza,
thus vaccination against influenza is unnecessary.
One month of prednisone with concomitant IVIG therapy (choice B) is not
indicated at this point. This patient has already failed these therapies and
her condition has improved after the splenectomy.
Permanent prednisone therapy and influenza vaccine (choice D) is an
inappropriate choice. Permanent prednisone would act to chronically
suppress this patient's immune system making her more susceptible to an
infection. Once again, influenza vaccination is not warranted in asplenic
patients.
Two weeks of prednisone therapy (choice E) is inappropriate at this point.
As stated before, prednisone therapy has already failed and the patient has
improved post splenectomy.
19
A 4-year-old boy is admitted to the hospital earlier in the day because of
fever, irritability, and erythema of the hands and feet for the past week. His
mother has been giving him aspirin to reduce his temperature. His physical
examination on admission showed a temperature of 39.7 C (103.4 F),
bilateral conjunctival injection, an enlarged right-sided cervical lymph node
(1.8-cm), fissured lips, a red tongue with red papillae, pharyngeal hyperemia,
erythematous and edematous palms and soles, and a confluent, blanching
erythematous rash on the trunk. Intravenous fluids were started, the aspirin
therapy was continued, and laboratory studies were ordered. These
laboratory studies finally returned and showed an erythrocyte sedimentation
rate of 28mm/h and a platelet count of 490,000/mm3. The patient is extremely
uncomfortable and now shows desquamation of the fingers and toes. The
most appropriate therapy at this time is
A. corticosteroids
B. ibuprofen
C. intravenous gammaglobulin
D. oxacillin
E. penicillin V
Explanation:
The correct answer is C. This patient most likely has Kawasaki disease,
which is treated with aspirin and intravenous gammaglobulin. The disease is
characterized by a high fever for longer than 5 days, bilateral conjunctival
injection, fissured lips, a "strawberry tongue", mucosal change in the oral
pharynx, erythematous and edematous palms and soles with desquamation,
a polymorphous rash, cervical lymphadenopathy, an elevated erythrocyte
sedimentation rate, and thrombocytosis. The most important complication is
coronary artery aneurysms, which may be prevented by early treatment with
aspirin and intravenous gammaglobulin. An echocardiogram is necessary to
evaluate cardiac involvement.
Corticosteroids (choice A) are not used to treat Kawasaki disease. They may
be used to treat some of the symptoms in severe Henoch-Schonlein purpura
(HSP), which is a vasculitis involving kidneys, gastrointestinal tract, skin, and
joints. Individuals with HSP typically have a rash on the lower extremities
and buttocks.
Ibuprofen (choice B) is not the most appropriate therapy at this time. This
patient most likely has Kawasaki disease, which is treated with aspirin and
intravenous gammaglobulin.
Oxacillin (choice D) is used to treat staphylococcal scalded skin syndrome
(SSSS), not Kawasaki disease. SSSS is characterized by fever, malaise,
periorbital edema, and a generalized, fine, erythematous rash. With gentle
rubbing, the epidermal layer of the skin may exfoliate. It is usually preceded
by an upper respiratory infection. Thrombocytosis is generally not present.
20
Penicillin V (choice E) is the treatment for scarlet fever, not Kawasaki
disease. Scarlet fever usually presents with an erythematous sandpaper-like
rash with fever and a "strawberry" tongue. It is typically associated with
streptococcal pharyngitis. Desquamation may occur. It is usually not
associated with bilateral conjunctivitis, fissured lips, or thrombocytosis.
An 82-year-old with COPD, hypertension, and coronary artery disease is
brought to the emergency department by his daughter who reported that for
the last 30 minutes her father has been becoming progressively confused
and complaining of chest pain and shortness of breath. His temperature is
38.0 C (100.7 F), systolic blood pressure is 60 mm Hg, pulse is 133/min and
irregular, and respiratory rate is 24/min. His pulse oximetry reveals 92%.
Physical examination shows a confused elderly male in moderate respiratory
distress. He has bilateral rales and a tachycardic irregularly irregular heart
beat. Peripheral pulses are very weak. An electrocardiogram shows atrial
fibrillation with a rate of 180. No ST elevations are noted. The most
appropriate next step in this patient's management is to
A. administer adenosine, intravenously
B. admit him to the cardiac intensive care unit
C. begin a diltiazem drip with heparin
D. perform electrical cardioversion
E. transport him to the cardiac catheterization laboratory
Explanation:
The correct answer is D. This patient has an unstable supraventricular
tachycardia and therefore needs immediate synchronized cardioversion.
Synchronized cardioversion is indicated with tachycardia and hemodynamic
instability such as pulmonary edema, hypotension, altered mental status,
chest pain secondary to ischemia, or acute MI.
Adenosine (choice A) is indicated if you have a stable patient with a narrow
complex tachycardia and you are trying to determine the specific diagnosis.
It is not indicated in an unstable patient.
Cardiac intensive care unit (choice B) would certainly be the final destination
of this patient, but it would be irresponsible to transport an unstable patient.
Remember, unstable tachycardia is a "code" and needs to be managed
immediately, not after transport is arranged and completed.
Diltiazem with heparin (choice C) is indicated in patients with stable atrial
fibrillation. If this patient was stable and had no contraindication to
anticoagulation, this would be a reasonable first step.
Ischemia or MI might be the etiology of this tachyarrhythmia or they might be
the result of this arrhythmia. Cardiac catheterization (choice E) might be
21
indicated later.
A 79-year-old man with a history of colon cancer comes to the emergency
department because of right lower extremity pain below the knee, which is
worse with ambulation for the past 4 hours. He has never had a pain like this
before and rates the pain as 9/10 in intensity below the knee. He denies
shortness of breath. Physical examination is significant for absent dorsalis
pedis and posterior tibial pulses in the right lower extremity. The right foot and
leg is cold with 1+ edema on all aspects of the leg and foot. All other pulses
are 3+. Electrocardiogram demonstrates occasional premature ventricular
contractions at a rate of 79/min. An abdominal plain film is normal. The next
step in the management of this patient is to
A. initiate heparin therapy, intravenously
B. initiate heparin therapy, intravenously, and prepare him for a
surgical embolectomy
C. order an ultrasound of the left leg
D. prepare him for a surgical embolectomy
E. provide symptomatic treatment with oxycodone
Explanation:
The correct answer is B. This patient is likely having an acute arterial
occlusion leading to a cold, pulseless foot. This patient is at risk for arterial
insufficiency because of a history of malignancy. Treatment is emergent and
consists of immediate heparin and surgical embolectomy.
Heparin (choice A) will stop further clots from forming, but it will not dissolve
the clot that is causing this limb-threatening circulatory compromise.
This patient is likely having an acute arterial occlusion leading to a cold,
pulseless foot. This patient is at risk for arterial insufficiency because of a
history of malignancy. Treatment is emergent and consists of immediate
heparin and surgical embolectomy. An ultrasound (choice C) should only be
attempted in a non-emergent setting, such as if pulses were still present, but
reduced or normal with complaints of pain.
As surgical embolectomy (choice D) in this patient is not sufficient, heparin
therapy must also be started to avoid further clot formations in this oncology
patient.
Symptomatic treatment alone (choice E) will likely lead to the loss of the
threatened limb. This patient is likely having an acute arterial occlusion
leading to a cold, pulseless foot. This patient is at risk for arterial
insufficiency with her history of malignancy. Treatment is emergent and
consists of immediate heparin and surgical embolectomy.
22
A 5-year-old boy is brought to the clinic for a periodic health maintenance examination.
He is generally healthy, enjoys school, plays well with his siblings and with other children
his age. He and his family live in a housing development down the street that was built
10 years ago. Since his mother usually works until late in the evening, he tends to spend
a lot of time at a friend's apartment in an old, dilapidated housing development nearby.
You notice that he has unusually pale skin and mucus membranes and so you inquire
about related symptoms. The mother tells you that she has noticed that he is
significantly more tired than his siblings and he has been a "bit irritable" lately but she
"didn't think nothing of it." He is up-to-date on all of his immunizations. There is no family
history of blood disorders, however several of his playmates "are anemic." You decide to
order hemoglobin, hematocrit, and a peripheral blood smear and schedule a follow-up
visit in 1 week. He returns for his next appointment and you review the results of the
laboratory studies. His hemoglobin is 9.5 g/dL, hematocrit is 30%, and the peripheral
blood smear shows microcytic red blood cells with basophilic stippling. The most
appropriate next step is to
A. administer ferrous sulfate, orally
B. administer dimercaprol, orally
C. administer edetate disodium, orally
D. determine B12 levels
E. determine blood lead levels
F. obtain an abdominal radiograph
G. order hemoglobin electrophoresis
Explanation:
The correct answer is E. This patient has a microcytic anemia with basophilic stippling.
Given that he spends a lot of time in an old, dilapidated housing development, you need
to consider lead poisoning as a diagnosis. It is not uncommon for children who live or
frequently visit houses that were built before 1950, or live or frequently visit houses that
were built before 1978 that are undergoing renovations, to develop elevated lead levels.
Lead paint is the most common culprit in these cases. The case also tells you that he
has several playmates that are anemic, which is also consistent with lead poisoning if
they are spending time in the same house or apartment complex. Lead has been
removed from paint and gasoline over the years to try to decrease the number of cases
of lead poisoning. Lead levels over 10 mg/dL are considered abnormal.
Administer ferrous sulfate, orally (choice A) is the correct answer for a patient with iron
deficiency anemia, not for lead poisoning, which this patient may have. Basophilic
stippling in the peripheral blood smear is not typically associated with iron deficiency
anemia and therefore further studies need to be performed to determine the cause of
this patient's anemia.
Dimercaprol (choice B) and edetate disodium (choice C) are used to treat established
cases of lead poisoning. Environmental interventions are usually appropriate for lead
levels under 25 mg/dL, while a combination of environmental interventions and oral
chelation therapy is usually considered for children with lead levels between 25 and 44
23
mg/dL. Intramuscular or intravenous dimercaprol is typically used for patients with lead
levels above 70 mg/dL and intramuscular or intravenous edetate disodium is used for
patients with lead levels between 45 and 69 mg/dL. Before starting any treatment, blood
lead levels need to be determined.
Determining B12 levels (choice D) is appropriate if this patient had a macrocytic anemia,
not a microcytic anemia.
Obtaining an abdominal radiograph (choice F) is usually recommended before beginning
chelation therapy for lead poisoning to look for enteral lead, which if present may also
require bowel decontamination. You should determine blood lead levels before getting
the radiograph.
Hemoglobin electrophoresis (choice G) is not the next best step because while
thalassemia may be associated with basophilic stippling, there is no family history of
blood disorders, which makes this less likely than lead poisoning. If lead levels are
normal, hemoglobin electrophoresis should be considered.
A 54-year-old woman that you have been treating for hypertension comes to
the office for a "blood pressure check." She tells you that she recently
stopped taking the enalapril that you prescribed because of the "annoying"
side effects. A friend of hers, who is also hypertensive, told her about an
herbal therapy that has "done wonders" for his blood pressure. The patient
says that she was a bit hesitant at first, but that she has been taking it for
about 3 months now, and she feels great. You review her chart and note that
her blood pressure has been ranging from 120/80 to 130/80 mm Hg over the
past year. Today, her blood pressure is 150/90 mm Hg and pulse is 70/min.
Physical examination is otherwise unremarkable. You should advise her that:
A. Her blood pressure is still elevated, and that the herbal therapy is
not only doing her "no good," but it may actually lead to dangerous
complications
B. It is difficult for you to continue treating her if she is just going to
discontinue the medications that you prescribe
C. She should consider going to another physician who will
incorporate herbal remedies with medications
D. She should have called you when she decided to discontinue the
enalapril
E. You can give her a different medication, without the "annoying" side
effects of enalapril, that she can take instead of the herbal therapy
Explanation:
The correct answer is E. This patient has already stopped taking the
enalapril and started taking herbal therapy, so the best thing that you can do
at this time to maintain a good physician-patient relationship is offer another
medication for her hypertension. Many patients make use of alternative
medicine practices and it is important for you to be understanding, or your
patients will not tell you what they are using. It is best to dissuade them from
24
questionable practices, not by criticizing the practices, but by recommending
better alternatives. In this case, enalapril is often associated with a cough,
and so you should recommend another anti-hypertensive agent without this
side effect.
Telling her that her blood pressure is still elevated and that the herbal
therapy is not only doing her "no good," but that it may actually lead to
dangerous complications (choice A) is not the best choice in order to
maintain a good physician-patient relationship. You should recommend a
better alternative as opposed to criticizing the treatment that she is taking.
Criticism may make the patient defensive and as a result, the patient may
not heed your recommendations. If she does not respond to your
recommendations, it is necessary to point out that her blood pressure is still
elevated and that you are concerned. If she still refuses, it is probably best
to document this in her chart.
You should not tell her that it is difficult for you to continue treating her if she
is just going to discontinue the medications that you prescribe (choice B).
Patients ultimately make their own medical decisions and it is the physician's
job to provide them with all the relevant material to make informed decisions.
In this case, you should be as respectful as possible about her use of
alternative therapies, but try to dissuade her by recommending better
alternatives. Over 1/3 of all patients make use of alternative medicine at this
time and if you are not respectful and understanding, you will have very few
patients.
At this point you should not tell her that she should consider going to another
physician who will incorporate herbal remedies with medications (choice C).
Since you ultimately want her to discontinue the herbal therapy and begin
another antihypertensive medication, you should be respectful and offer her
alternatives, without criticizing her choices. Since she is your patient, it is
your responsibility to provide her with all relevant information, and respect
her decisions, even if you do not agree with them. You want to maintain a
physician-patient relationship, so you should try to work with her.
Telling her that she should have called you when she decided to discontinue
the enalapril (choice D) will only make her defensive, because the action is
already in the past. It does not address future solutions.
A 59-year-old woman with acute congestive heart failure is admitted to the
intensive care unit. She was transferred from the medical floor where she
was found to be in florid pulmonary edema with hypoxemia and respiratory
distress. She was intubated by the anesthesia airway team at that time of
transfer. She was transferred to the medical intensive care unit for aggressive
diuresis and ventilator management. On arrival to the unit, it is determined
that the patient will require frequent arterial blood samplings to monitor her
ventilation status. A decision is made to place an indwelling arterial catheter
for this purpose. The artery that carries with it the highest risk for
complications when used for arterial cannulation is the
25
A. brachial artery
B. dorsalis pedis artery
C. femoral artery
D. radial artery
E. ulnar artery
Explanation:
The correct answer is A. A concern when indwelling arterial catheters are
placed is that the vessel thromboses. If this occurs, perfusion to the tissues
distal to the thrombosis depends upon collateral flow. The brachial artery is
an end artery and its collaterals are distal to the insertion site in the
antecubital fossa. Therefore, if it were to thrombose, there would be no
inflow to the forearm.
The dorsalis pedis (choice B) is found on the dorsum of the foot and has
extensive collateral flow via the posterior tibial artery.
The femoral (choice C) is located in the anterior leg at the floor of the
femoral triangle and is sub-served by many collaterals.
The radial (choice D) and the ulnar (choice E) are the most common sites for
insertion of an arterial catheter. These two arteries are collaterals of one
another and it is very rare that the ulnar (the less used artery for
cannulation) is not patent.
A 27-year-old man comes to the emergency department because of
increasing fatigue, malaise, chills, and low-grade fevers over the last 2
weeks. He reports no recent sick contacts and denies any significant past
medical history. The patient does mention that he uses heroin frequently but
not since last week. His temperature is 38.8 C (101.8 F), blood pressure is
85/60 mm Hg, and heart rate is 120/min. On physical examination, the patient
appears gaunt, malnourished, and dehydrated. A faint systolic murmur is
audible on cardiac auscultation. Needle tracks are found at both antecubital
fossa. Petechiae are noted across his back and splinter hemorrhages are
found under the nail beds of his right hand. Laboratory studies show:
A chest radiograph shows normal lungs and cardiac silhouette. An
electrocardiogram reveals sinus tachycardia. Urinalysis shows 2+ proteinuria,
3+ red blood cells, and 1+ ketones. The patient is admitted to the hospital
where he becomes progressively more confused and disoriented. Three sets
26
of blood cultures are drawn and intravenous fluids are initiated. The most
appropriate next step in management is to
A. begin nafcillin and gentamicin, intravenously
B. begin penicillin and clindamycin, intravenously
C. obtain an immediate evaluation by cardiac surgery
D. order an urgent transesophageal echocardiography
E. start methadone therapy
Explanation:
The correct answer is A. This case describes a patient with acute bacterial
endocarditis that rapidly deteriorates soon after his initial presentation. He
presents with vague medical complaints and a history of intravenous drug
use, namely heroin. The initial medical symptoms and vital signs can be
consistent with opiate withdrawal. However, his physical exam reveals a
murmur, truncal petechiae, and splinter hemorrhages that are indicative of
severe endocarditis. In addition, he has an elevated white blood cell count.
The urinalysis and elevated creatinine indicate possible glomerulonephritis
from immune-complex deposition. Last of all, he has a notable change in his
mental status. All these data support the diagnosis of acute bacterial
endocarditis. Due to his rapidly worsening clinical course, it is imperative to
begin empirical antibiotic treatment once blood cultures are drawn, since
early antibiotic therapy may prevent further morbidity and sepsis. In
intravenous drug users, the most common pathogen is Staphylococcus
aureus. Other organisms include Staphylococcus epidermidis,
Streptococcus viridans, Enterococcus, and various Gram-negative rods.
Since the organism is unknown, broad antibiotic coverage is indicated.
Antibiotic therapy with nafcillin and gentamicin is a typical combination for
native valve endocarditis. If MRSA is prevalent, vancomycin and gentamicin
are an alternative combination.
Penicillin and clindamycin (choice B) are excellent antibiotics for Grampositive organisms, particularly the Streptococcus species and anaerobic
bacteria. However, Staphylococcus aureus is the predominant organism that
causes acute bacterial endocarditis in intravenous drug users, and neither
antibiotic covers Staphylococcus aureus well.
Cardiac surgery consultation (choice C) to evaluate for valvular replacement
surgery is indicated only if there is invasive infection, fungal infection,
recurrent systemic emboli, refractory infection, or refractory heart failure
present. Further evaluation of the patient will be required to determine if this
is necessary.
An echocardiogram (choice D) is required for definitive diagnosis and for
further assessment of the valve to rule out paravalvular abscesses or
invasive infection. However, this patient is unstable and there is enough
clinical evidence to support the diagnosis, so obtaining an echocardiogram
will only confirm diagnosis and should not delay treatment with antibiotics. It
27
is usually done after blood cultures are obtained and they are positive.
Methadone therapy (choice E) should be instituted for patients with opiate
withdrawal symptoms. However, this patient's acute condition is due to
bacterial endocarditis.
A 57-year-old woman with diabetes and nephrolithiasis is admitted to the
medical services for evaluation of her chest pain that began when playing
with her grandchild. She had a previous myocardial infarction and is statuspost a three-vessel bypass two years prior. Her medications include atenolol,
lisinopril, allopurinol, and atorvastatin daily. You are called to the patient's
room because the patient is currently complaining of chest pain. She reports
that while talking on the telephone, she became very angry with her daughter
and developed chest pain. On arrival she is lying in bed and appears
uncomfortable. She is diaphoretic and appropriately anxious. Her blood
pressure is 190/110 mm Hg and pulse is 110/min. She has an S4 gallop and
scant bi-basilar rales. An electrocardiogram shows sinus tachycardia with a
left axis deviation. Voltage criteria are met for LVH and there are ST segment
depressions of 2.5mm in leads V1-V5. The most appropriate next step in
management is to
A. administer verapamil, intravenously
B. give furosemide, intravenously
C. give morphine, intravenously
D. give nitroglycerin, intravenously
E. obtain a chest radiograph
Explanation:
The correct answer is D. This patient is having cardiac ischemia in the
setting of increased myocardial oxygen demand. She has known coronary
disease and her vital signs at the time of her angina episode show
hypertension and tachycardia. The patient must have her blood pressure
lowered acutely to the goal of terminating the angina with nitroglycerin.
Verapamil (choice A) is a calcium channel blocker that is used most often to
slow the ventricular rate in patients with atrial fibrillation. These drugs are
generally contraindicated in an acute myocardial ischemia as they tend to be
associated with worse outcome.
Furosemide (choice B) may work to relieve some pulmonary congestion that
this patient has as a result of her ischemia and increased left-sided filling
pressures. It however fails to address the primary problem of her increased
demand.
Morphine (choice C) is usually given to relieve pain, but nitroglycerin is more
important at this time because it can relieve pain and also reduce the
28
underlying ischemia.
A chest radiograph (choice E) would not be without benefit in this emergent
situation, it has no role in altering any decision making or therapeutic
intervention. This patient must have her blood pressure and heart rate
lowered.
A 5-year-old boy is brought to the clinic because of a fever for 5 days and a sore throat
and malaise. The mother tells you that he is usually a very healthy child and he is up to
date on all of his immunizations. Besides the mother, he lives at home with an older
brother and sister, neither of whom are sick. His temperature is 39.5 C (103.1 F). On
examination, he has a peeling rash on his extremities, one 2 cm lymph node on the right
anterior cervical chain, a confluent truncal rash, and mild conjunctivitis. Appropriate
management is taken. The most important long-term management of this child is
A. antibiotic prophylaxis to prevent rheumatic fever
B. echocardiograms to look for coronary artery aneurysms
C. excisional biopsy and surveillance of lymph nodes for malignancy
D. nothing, as this is a case of scarlet fever and he will completely recover
E. serial lumbar punctures
Explanation:
The correct answer is B. The vignette demonstrates a case of Kawasaki disease which
is characterized by high fever for >5 days, unilateral cervical lymph node, macular
papular rash to truck, peeling of hands and feet, and conjunctivitis. The most important
sequela of Kawasaki is the development of coronary aneurysms, and so children need
to be monitored with EKGs and echocardiograms for at least 2-3 months after the acute
illness. Aneurysms form from 7-45 days after illness onset.
Streptococcal pharyngitis is characterized by erythema of the oropharynx, palatal
petechiae, and cervical lymphadenopathy. There is no peeling, rash or conjunctivitis.
Antibiotics are used for treatment and rheumatic fever prophylaxis (choice A).
The enlarged lymph node is a reactive lymphadenopathy and need not be biopsied or
excised (choice C). Upon proper treatment for KD, lymphadenopathy regresses.
This is not a case of scarlet fever (choice D) which is characterized by exanthem and a
fine, sandpapery rash, and is usually caused by Streptococcus.
Although children with KD often present with a toxic appearance, lumbar punctures
(choice E) is not the standard of care and need not be performed.
A 74-year-old man comes to the office for a periodic health maintenance examination.
He has diet-controlled diabetes mellitus, hypertension, glaucoma, mild peripheral
vascular disease, and osteoarthritis. His medications include lisinopril, atenolol, aspirin,
29
and acetaminophen. He has smoked one pack of cigarettes per day for the past 35
years. His blood pressure is 160/80 mm Hg and pluse is 61/min. He has a left carotid
bruit and a 2/6 systolic ejection murmur heard best at the left sternal border. His
abdomen is soft with no masses but there is a previously appreciated abdominal bruit.
Concerning the patient's cardiac murmur, the most appropriate diagnostic test to further
evaluate its significance and severity is
A. a chest radiograph
B. a coronary catheterization
C. an echocardiogram
D. an electrocardiogram
E. an exercise treadmill test
Explanation:
The correct answer is C. The patient has a systolic ejection murmur heard best over the
aortic area. Although we are given no information of the radiation of this murmur, it
sounds to be a murmur of aortic stenosis. This is a serious condition for which early
correction is critical to avoid long-term complications such as congestive heart failure
and sudden cardiac death. A transthoracic echocardiogram is the appropriate imaging
modality to assess the patency of this or any valve.
A chest radiograph (choice A) gives no information about the valvular or tissue
architecture of the heart other than its gross size.
Coronary catheterization (choice B) is the gold standard test used to visualize the
coronary anatomy and is also used to further define previously diagnosed valvular
disease. For example, if this patient is found to have aortic valve disease, a coronary
catheterization will determine the transvalvular gradient and area of stenosis more
accurately that an echo.
An electrocardiogram (choice D) gives only aggregate electrical signal data about
aggregate electrical activity in the heart. It gives no direct or even indirect information
about valve anatomy.
An exercise treadmill test (choice E) is used as a surrogate test instead of angiography
to test for cardiac ischemia. A positive test usually implies a significant anatomical
coronary lesion and a negative test implies a non-significant lesion; thereby negating
the need for invasive angiography in all patients.
A 58-year-old man is admitted to the hospital directly from your office
because of new-onset atrial fibrillation. He has no significant past medical
history and does not take any medications. He is allergic to penicillin, to
which he develops a rash. In the office, his blood pressure was 109/78
mmHg, pulse was 99/min, and respiratory rate was 22/min. An
electrocardiogram demonstrated atrial fibrillation. Physical examination was
remarkable for an irregularly irregular cardiac rhythm. While you are
30
requesting further laboratory studies after arriving at his bedside the patient
slumps over and becomes unresponsive. You immediately look to the
continuous cardiac monitor and note that his heart rate is now 160/min and
he is still in atrial fibrillation. His pulse is barely palpable. The most
appropriate next step in management is to
A. perform asynchronous cardioversion
B. perform synchronous cardioversion
C. order an immediate transesophageal echocardiogram
D. schedule immediate cardiac catheterization
E. schedule immediate placement of an intraaortic balloon pump
Explanation:
The correct answer is B. The treatment of unstable atrial fibrillation (atrial
fibrillation associated with hypotension, myocardial ischemia, congestive
failure, etc.) is immediate synchronous cardioversion, in an attempt to
immediately restore sinus rhythm.
Performing an asynchronous cardioversion (choice A) is not considered the
first choice in the treatment of atrial fibrillation since it is not synchronized
with the QRS complex, and therefore runs the risk of providing electrical
energy during a time when the ventricle is at risk for developing more
dangerous dysrhythmias such as ventricular fibrillation.
Requesting an immediate transesophageal echocardiogram (choice C) is
incorrect since it will not provide any information useful for the emergent
treatment of unstable atrial fibrillation. Transesophageal echocardiography
can however play a role in ruling out atrial thrombi prior to anticoagulation in
patients with stable atrial fibrillation in preparation for cardioversion.
Scheduling immediate cardiac catheterization (choice D) is incorrect since
cardiac catheterization plays no role in the emergent management of
unstable atrial fibrillation. It is, however, the most appropriate intervention in
the setting of an acute ST elevation myocardial infarction.
Placing an intraaortic balloon pump (choice E) is incorrect since the
intraaortic balloon pump has no role in the management of heart failure in
the setting of a primary dysrhythmia. An intraaortic balloon pump can
conversely play a significant role in the management of heart failure due to
myocardial ischemia.
A 45-year-old woman comes to the emergency department with swelling in
her left leg. She had been vacationing in Australia, and had noticed the
swelling since her airplane ride back to the U.S. two days ago. She has also
noted some erythema over the affected area, but denies any fevers or chills.
She also denies any trauma to the leg. Her medications include oral
contraceptive pills and ranitidine. Her temperature is 37 C (98.6 F), pulse is
31
80/min and regular, blood pressure is 120/80 mm Hg, and respiratory rate is
18/min. She denies tobacco use. On exam, you note that her jugular venous
pressure is 7. She has 2+ edema of her left lower extremity extending to her
mid thigh, mild overlying erythema without increased warmth, and calf pain
with dorsiflexion of the left foot. There is no evidence of streaking or trauma
and no palpable cords. The pulses on the affected extremity are within
normal limits. Her white blood cell count is 8,000/mm3, hematocrit is 38 %,
and platelet count is 286,000/mm3. The most likely diagnosis is
A. arterial thrombus
B. cellulitis
C. congestive heart failure
D. deep vein thrombosis
E. lymphangitis
Explanation:
The correct answer is D. The presence of unilateral edema without evidence
of infection (fever, chills, warmth over the affected area, elevated white
blood cell count) in the setting of a recent history of decreased activity (long
airplane ride) all suggests this diagnosis. The presence of a positive Homan
sign (calf pain with dorsiflexion of the foot) is also suggestive of a deep vein
thrombosis.
An arterial thrombus (choice A) is unlikely given that the pulses on the
affected limb are not diminished, there is no evidence of pallor or coolness
of the affected limb, and there is no history of pain or paresthesias of the
limb.
Cellulitis (choice B) is less likely given that there is no history of fever or
chills, and the affected limb is without any increase in warmth despite the
slight erythema, and the white blood cell count is not elevated.
Congestive heart failure (choice C) is unlikely since there is no history of
congestive failure or coronary artery disease, and the patient has unilateral
edema (the edema associated with congestive failure is typically bilateral).
Lymphangitis (choice E) is unlikely since there is no evidence of infection on
physical exam (including streaking over the affected limb which would be
classic for lymphangitis) or laboratory evaluation.
A 67-year-old man comes to the office because he "just can't do as much walking
anymore" and he has increasing episodes of "pounding heart beats." He used to go
walking with his wife at the local high school, but he has been feeling "too tired" to keep up
with her over the last few months. He is a new patient and reluctantly admits that he has
not been to a physician in "decades". He says that he has led a "vibrant and healthy" life
up until now. His temperature is 37 C (98.6 F), blood pressure is 110/80 mm Hg, and
pulse is 87/min. Physical examination shows pale skin and mucous membranes.
32
Laboratory studies show:
The most appropriate initial management is to
A. administer a trial with oral ferrous sulfate
B. do a fecal occult blood test
C. refer him for a colonoscopy
D. schedule him for an air-contrast barium enema
E. treat him with vitamin B12, subcutaneously
Explanation:
The correct answer is B. This patient has iron deficiency anemia, which in a man of his
age, must be assumed to be due to chronic blood loss until proven otherwise. The most
important first step is to identify the source of bleeding. In men, gastrointestinal bleeding
is the most common underlying cause of iron-deficiency anemia. A fecal occult blood test
is the initial screening test in this situation. If positive, proper investigations must be
carried out to determine the source, such as a peptic ulcer, gastritis, or colon cancer.
A trial with oral ferrous sulfate (choice A) is inappropriate because, while this is the
treatment for iron-deficiency anemia, the underlying condition that is causing it must be
identified.
A colonoscopy (choice C) and a barium enema (choice D) may be necessary later on in
the work-up. However, a test for occult blood is the initial screening test.
It would be inappropriate to treat him with vitamin B12, subcutaneously (choice E)
because this is the treatment for megaloblastic anemia secondary to pernicious anemia.
In contrast to iron-deficiency anemia, megaloblastic anemia is characterized by
macrocytes (MCV >110 fL).
A 21-year-old man comes to the clinic for a follow-up visit. He has been found
to be hypertensive with bilateral arm blood pressures in the 140-160/100-110
mm Hg range on several visits. The patient has no past medical history and
has no complaints. Review of systems is significant for occasional “band-like
headaches". Physical examination is normal including a normal retinal
examination. Urinalysis is normal. An electrocardiogram demonstrates normal
sinus rhythms at a rate of 75/min, with mild left ventricular hypertrophy. His
blood pressure is now 150/100 mm Hg after treatment with maximal doses of
atenolol, hydrochlorothiazide, and captopril. The next step in the evaluation of
33
this patient's hypertension is to
A. order a duplex ultrasound of the renal arteries
B. order an echocardiogram
C. prescribe triamterene
D. repeat the electrocardiogram
E. repeat the ophthalmoscopic examination
Explanation:
The correct answer is A. This patient should be suspected of having renal
artery stenosis, most likely secondary to fibromuscular dysplasia from the
early age of onset of severe hypertension. In addition, the poor control of
this patient's hypertension, even with three agents is suspicious for
renovascular hypertension. About 5% of patients with hypertension have
renovascular hypertension. The renal arteries need to be evaluated by
duplex ultrasound of the renal arteries, conventional angiography, spiral
computed tomography, or magnetic resonance angiography.
Given signs of left ventricular hypertrophy on electrocardiogram,
echocardiography (choice B) is warranted. The next immediate step
however, is an evaluation of the renal arteries.
Triamterene (choice C) is a potassium-sparing diuretic often used in
conjunction with hydrochlorothiazide. It is very unlikely that the three agents
prescribed in high doses could not control the blood pressure of a patient
with essential hypertension. Renovascular hypertension must be suspected.
Repeat electrocardiograms (choice D) should be performed every year to
evaluate for the development of end-organ damage in a chronic
hypertensive. This patient has electrocardiographic evidence of left
ventricular hypertrophy. However, it is best followed with echocardiography.
Repeat ophthalmoscopy examinations (choice E) should be performed every
year or so to evaluate for the development of end-organ damage in a
chronic hypertensive.
A 41-year-old woman who you have been treating for depression, anxiety,
and dysmenorrhea comes to the office because she "has the flu." She
describes having low-grade fevers, chills, a sore throat, and rhinorrhea for the
past 3 days. She also reports multiple sick contacts. She is a schoolteacher
and lives with her husband and 3 children. Her temperature is 37.0 C (98.6
F), blood pressure is 120/65 mmHg, pulse is 102/min, and respirations are
23/min. Physical examination is unremarkable except for some pallor of her
mucous membranes. A laboratory evaluation reveals a hematocrit of 28%
and normocytosis. The most appropriate next laboratory test to order is
34
A. hemoglobin electrophoresis
B. reticulocyte count
C. serum ferritin level
D. serum folate level
E. serum iron level
Explanation:
The correct answer is B. With a normocytic anemia, the first diagnostic
maneuver should be to decide whether it is a result of blood loss or marrow
failure, which are the two primary causes of a normocytic anemia. A
reticulocyte count will help with this since an appropriately elevated count
will suggest ongoing loss, while a depressed count would suggest marrow
failure.
Hemoglobin electrophoresis (choice A) is reserved for the further evaluation
of anemias suspected to result from hemoglobin anomalies such as sickle
cell disease. There is no data to suggest such pathology here.
A serum ferritin level (choice C), a measure of total body iron stores, would
not be useful in the setting of a normocytic anemia since an iron deficiency
anemia is associated with microcytosis.
A serum folate level (choice D) would not be useful in the setting of a
normocytic anemia since folate deficiency is associated with macrocytosis.
A serum iron level (choice E) would not be useful in the setting of a
normocytic anemia since iron deficiency anemia is associated with
microcytosis.
A 48-year-old man is admitted to the hospital because of a 2-hour history of chest pain
and shortness of breath that came on suddenly when he was shoveling snow from the
walkway to his house. His electrocardiogram on admission showed ST elevation that
started to descend as the T waves inverted and Q waves appeared. He is treated with
streptokinase, aspirin, intravenous heparin, oxygen, nitroglycerin, metoprolol, and
morphine. His chest pain resolves and he is settled into the cardiac care unit. As the
days progress he is able to sit up, dangle his feet over the side of the bed, and begins to
ambulate in his room. On hospital day 6, you go to examine him before he is discharged
home. It is appropriate to advise the patient that
A. beta-blockers will decrease his morbidity, not his mortality, following a
myocardial infarction
B. bed rest is necessary for 1 week
C. maximal exercise stress testing should be performed in 2 days
D. sexual activity can be resumed in 2 to 4 weeks
35
E. supervised gradual aerobic program should be started in 2 months
Explanation:
The correct answer is D. After a myocardial infarction, normal sexual activity can
typically be resumed 2-4 weeks after discharge from the hospital.
It is inaccurate to tell him that beta-blockers will decrease his morbidity, not his
mortality, following a myocardial infarction (choice A) because they have been shown to
decrease mortality, reinfarction, and the risk of sudden death.
Bed rest is necessary for 1 week (choice B) is wrong because bed rest is only
recommended for the first 12 hours after a myocardial infarction, and after that, the
amount of activity should be gradually increased from sitting up, to dangling the feet, to
walking around their room, then the hall. By the time they are discharged, 5-7 days
later, walking is important, and a moderate exercise program should be encouraged
several weeks later. It is necessary to evaluate the patient's exercise tolerance a couple
of weeks following a MI.
Maximal exercise stress testing should be performed in 2 days (choice C) is incorrect. A
submaximal exercise test may be performed in the hospital before discharge in stable
patients, and a maximal exercise stress test can be performed in 2-4 weeks. This is to
assess the risk of another infarction.
A supervised gradual aerobic program should be started in 2 months (choice E) is
incorrect because it should be started after discharge, beginning with walking and then
increased as tolerated.
A 56-year-old active woman is brought to the hospital by her husband who is
a physician because of an episode of weakness and dizziness that occurred
suddenly during her aerobics class this morning. She states, “I felt like I was
suffocating and I almost passed out.” The episode lasted for approximately
15 minutes and was finally relieved after sitting down and resting. When she
got home and relayed the story to her husband, he immediately took her
blood pressure, which was 62/80mm Hg and pulse, which was 90/min. He
gave her an aspirin to chew and although she was initially reluctant, he
convinced her to come to the emergency department for evaluation. She tells
you that although she has never had similar episodes in the past, she has
been feeling a little more “winded” than usual while walking over the past 3
weeks. She feels much better now and tells you in confidence that she thinks
her husband is “just overreacting.” Her past medical history includes
hypertension, hypothyroidism, and high cholesterol. She is taking simvastatin,
levothyroxine, and amlodipine. She admits to an occasional cigarette when
her husband is not around and drinks one to two glasses of wine per day.
She works as a lawyer in a prestigious law firm. Her father died at age 42
from a massive myocardial infarction and her mother is alive and well. Her
blood pressure is 160/90 mm Hg and pulse is 100/min. Physical examination
is unremarkable. You place her on oxygen and order an electrocardiogram
that shows T wave inversions in leads V5 and V6 without any ST elevation or
depression. The most appropriate next step in management is to
36
A. administer beta blockers, heparin, and glycoprotein IIb/IIa inhibitors
and admit her to a monitored setting for serial electrocardiograms and
cardiac enzymes
B. administer tissue plasminogen activator (t-PA), 100mg IV over 90
minutes
C. consult a cardiologist and await advice concerning appropriate
management of this patient
D. order a bedside echocardiogram
E. order cardiac enzymes and if normal send her home with a referral
to a cardiologist within the next week
Explanation:
The correct answer is A. It is important to remember that women have
atypical presentations of cardiac disease including both unstable angina and
acute myocardial infarctions. This patient has multiple risk factors including
hypertension, high cholesterol, cigarette smoking, and a first degree relative
with an acute myocardial infarction at a young age. Even though her
symptoms have completely resolved upon presentation, she describes a
very concerning episode of weakness, shortness of breath, dizziness, and
nausea which likely represents cardiac ischemia. Additionally, the fact that
she has been “winded” lately may represent progressive angina. The history
taken together with the abnormal EKG finding of lateral T wave inversions
lead you to the diagnosis of unstable angina. The standard of care for the
management of high-risk patients with unstable angina includes beta
blockade with intravenous medications to block sympathetic nervous system
stimulation of heart rate and myocardial contractility, which thereby reduces
myocardial oxygen consumption as well as decreases the incidence of
ventricular ectopy and fibrillation. Heparin is also indicated to directly inhibit
thrombin and thus decrease thrombus propagation of unstable plaques.
Glycoprotein IIb/IIIa inhibitors act on the Integrin GP IIb/IIIa receptor on the
membrane of platelets which leads to the inhibition of platelet aggregation
and thus decreases clot formation. GP IIb/IIIa inhibitors are indicated for
those patients with high-risk unstable angina (those with multiple risk factors
who present with symptoms suggesting cardiac ischemia and have EKG
changes such as ST depression or T wave inversion). This patient must also
be admitted for continuous cardiac monitoring and have serial cardiac
enzymes (CPK, CK-MB, and troponin) with serial EKGs every 8 hours for a
total of 3 sets. Depending upon the resources of the hospital, many
cardiologists would promote rapid angiogram with possible angioplasty and
stent placement as necessary for this patient.
The administration of t-PA, tissue plasminogen activator (choice B) is
inappropriate in this patient as she does not meet the criteria for fibrinolytic
administration. The criteria for fibrinolytic therapy are chest pain suggestive
of cardiac ischemia for >20 minutes but less than 12 hours with ST segment
elevation of >1mm in 2 or more contiguous EKG limb leads, or ST elevation
of >2mm in 2 or more contiguous EKG precordial leads, or a new left bundle
branch block.
37
Consultation with a cardiologist is important (choice C). However, to defer
treatment until a patient is evaluated by a cardiologist is a mistake. Instead,
one should act immediately using the American Heart Association guidelines
for the management of acute coronary syndromes.
An echocardiogram of the heart (choice D) may be helpful in determining if
any wall motion abnormality exists. However, it is not the most important
next step in the management of this patient.
One set of cardiac enzymes at initial presentation (choice E) is insufficient in
the evaluation of any patient with a suspected acute coronary syndrome. In
myocardial infarction the CPKs rise within 4-6 hours and peak at 24-30
hours. The CK-MB rises within 3-4 hours and peaks between 12-24 hours.
The troponin will rise within 2-6 hours and peak within 12-24 hours. These
tests help to distinguish between unstable angina with no actual myocardial
cell death and non-Q wave myocardial infarctions.
A 67-year-old man comes to the office because of a 3-month history of
progressive left leg pain and cramping when walking. The pain is only present
during exercise and is relieved by rest. He suffered a myocardial infarction 2
years earlier. He smokes a pack of cigarettes a day and drinks a "couple of
beers" a day. He has a prescription for oral nitrates and a beta-blocker, but
admits that he never takes them. His blood pressure is 130/90 mm Hg and
his pulse is 75/min. Physical examination shows a diminished femoral pulse
and an absent popliteal and pedal pulses on the left leg as compared to the
right leg. The left leg has no hair below the knee, the skin is shiny and
smooth, and the toenails are thickened. Neurological examination is
unremarkable. The most appropriate next step is to
A. advise him to quit smoking and maintain meticulous foot care
B. determine the ratio of ankle to brachial arterial pressures
C. recommend a daily aspirin therapy
D. refer him to a surgeon for evaluation for surgical revascularization
E. schedule an angiography of the left leg
Explanation:
The correct answer is B. This patient seems to be describing intermittent
claudication, which is a sign of peripheral vascular disease. Patients often
complain of pain during exertion that is relieved by rest. The physical
findings in this case are very consistent with this diagnosis. Noninvasive
evaluation is usually recommended initially and consists of determining the
ratio of ankle to brachial arterial pressures. The ankle/brachial artery ratio is
determined by measuring and comparing the 2 blood pressures. If the ratio
is less than 1 (greater than or equal to 1 is considered normal), peripheral
38
artery disease is present.
After it is determined that peripheral artery disease is present, it is
appropriate to advise him to quit smoking and maintain meticulous foot care
(choice A). Additional treatment includes a gradual exercise program and
possible aspirin or other antiplatelet drugs. Surgical revascularization is
usually considered for severe, disabling disease.
It may be appropriate to recommend daily aspirin therapy (choice C) after
you have determined that peripheral artery disease is present to reduce the
risk of cardiovascular events.
It is inappropriate to refer him to a surgeon for evaluation for surgical
revascularization (choice D) at this time. You should first perform a
noninvasive evaluation before this is even considered.
It is inappropriate to schedule angiography of the left leg (choice E) at this
time. You should first perform a noninvasive evaluation before this is even
considered. An angiography is not used as a routine diagnostic study for the
initial evaluation of claudication. It should only be performed if surgical
revascularization is being considered.
You are seeing a 33-year-old man who has a history of idiopathic
hypertrophic subaortic stenosis for a follow-up visit in your clinic. His
medications include metoprolol and ranitidine. He smokes 1 pack per day and
drinks a 6–pack of beer each weekend. He has a family history of sudden
death. His blood pressure is 110/67 mmHg and pulse is 78/min. There is a
III/VI systolic ejection murmur at his cardiac base. You decide to repeat his
cardiac examination with handgrip. Following this maneuver, you would
expect to hear
A. a decrease in his systolic ejection murmur
B. a holosystolic murmur over the cardiac apex
C. an increase in his systolic ejection murmur
D. a mid-systolic "click"
E. a "mill wheel" murmur over the precordium
Explanation:
The correct answer is A. Idiopathic hypertrophic subaortic stenosis (IHSS) is
characterized by an aortic outflow obstruction (primarily from the anterior
mitral valve apparatus). The degree of obstruction is dependent on multiple
factors, including afterload. By increasing afterload with handgrip one
reduces the amount of ventricular ejection, increasing end-systolic
ventricular size. This reduces the amount of outflow obstruction and
decreases the systolic murmur of IHSS.
A holosystolic murmur over the cardiac apex (choice B) is characteristic of
39
mitral regurgitation which is not seen with idiopathic hypertrophic subaortic
stenosis.
An increase of the systolic murmur (choice C) does not occur with handgrip
(or other maneuvers that increase afterload) with idiopathic hypertrophic
subaortic stenosis as discussed above.
A mid-systolic "click" (choice D) is seen with mitral valve prolapse, but is not
characteristic of idiopathic hypertrophic subaortic stenosis.
A "mill wheel" murmur over the precordium (choice E) can be heard with
large air embolism.
An 18-month-old boy is brought to the office because his mother claims that
he appears very pale but is otherwise acting normally. She tells you that he
drinks approximately 48 ounces of whole milk per day. There is no history of
anemia in the family. His temperature is 37.0 C (98.6 F), blood pressure is
80/50 mm Hg, pulse is 120/min, and respirations are 20/min. Physical
examination shows pallor of the skin and mucous membranes. A 2/6 systolic
ejection murmur is noted. Laboratory studies show:
The most appropriate initial management is to
A. evaluate the family for anemias
B. give folate, orally
C. give vitamin B12, intramuscularly
D. recommend that the patient cut down on the milk intake and start
oral iron supplementation
E. transfuse with whole blood
Explanation:
The correct answer is D. Iron deficiency is the most common nutritional
deficiency in children ages 9-15 months. It can be due to the decreased
availability of dietary iron, impaired absorption of iron secondary to frequent
infections, or an increased requirement of iron for growth. Also, iron
deficiency caused by high cow's milk intake is a relatively common problem
in young children. Frequently, parents will continue giving large amounts of
milk to their children because they are used to giving as much formula as
40
the child wants during the first year of life. This causes a few problems. First,
it will cause the child to eat less regular food and therefore not receive the
vitamins and minerals necessary for growth. In addition, as seen in this
case, iron deficiency can occur. Milk doesn't contain much iron and through
mechanisms not well understood, it also inhibits iron absorption. The anemia
can be profound in many cases and may require transfusion. In this case the
signs of iron deficiency are seen in the very low MCV, high RDW, and
anemia. Since this is a chronic process, the anemia is well tolerated by the
child and therefore he is not symptomatic. Therefore he just requires iron
supplementation and a decrease in the daily milk intake to around 24
ounces per day. With oral iron a prompt reticulocytosis and rise in
hemoglobin and hematocrit will be seen.
At this point an expensive evaluation of the family for anemias (choice A) is
not warranted because the source for the anemia seems apparent. If after a
few months of iron and a decrease in milk intake the RBC indices and Hct
do not improve, then further investigation in necessary.
Folate deficiency would cause a macrocytic anemia and is not a
consequence of excessive cow's milk intake. Folate deficiency can be seen
in children who are raised on goat milk. Giving folate (choice B) to this
patient is inappropriate.
As with folate deficiency, B12 deficiency causes a macrocytic anemia that is
not seen in this case and is not very common in young children with no other
illnesses or deficiencies. Therefore giving B12 (choice C) to this patient is
inappropriate.
Transfusion (choice E) is not warranted here because the patient, although
pale, is asymptomatic. In addition, children are usually transfused with
packed RBCs to restore hemoglobin not whole blood.
A 78-year-old woman is admitted to the hospital because of a fever,
productive cough, and a chest x-ray demonstrating right lower lobe
consolidation. Her past medical history is significant for seasonal allergies.
She has been taking estrogen/progesterone replacement since menopause
19 years ago and occasional acetaminophen for headaches. The patient lives
alone at her home and she does not drink alcohol or smoke. Review of
systems is significant for weakness attributed to “old age". On the day prior to
discharge, a repeat chest x-ray shows the pneumonia to be resolving. An
incidental note is made of severe osteoporosis involving all of the bones
visualized on the film. Vital signs are temperature 38.8 C (101.8 F), blood
pressure 100/50 mm Hg, pulse 90/min, and respirations 10/min. Physical
examination is significant only for decreased breath sounds at the right lung
base. The patient is neurologically intact and wants to return home.
Laboratory studies show a leukocyte count 15,000/mm3, hematocrit 28%, and
platelets 150,000 mm3. The next step in the management of this patient is to
A. discharge her and do a bone marrow biopsy as an outpatient
41
B. discharge her and send her for a bone scan as an outpatient
C. discharge her and order serum protein electrophoresis as an
outpatient
D. do a bone marrow biopsy before discharge
E. order a bone scan and serum protein electrophoresis before
discharge
Explanation:
The correct answer is C. This patient is presenting to the hospital with
pneumonia. The findings of diffuse osteoporosis in a patient on hormone
replacement therapy are suspicious for a multiple myeloma. The clinical
signs are nonspecific constitutional symptoms such as general malaise or
weakness. Laboratory data includes anemia, an elevated creatinine from
secondary renal dysfunction, and elevated IgA and IgG levels. The first
priority is to treat the patient for pneumonia and return the patient to her
normal life as soon as possible. The next priority is to evaluate for multiple
myeloma and the family of disorders involving monoclonal proliferation. This
would include a serum protein electrophoresis (SPEP), a plain radiographic
bone survey, and an assessment of renal function. Patients that are
asymptomatic with no significant anemia, pathological fractures, or renal
failure do not require treatment.
A bone marrow biopsy (choice A) is sometimes necessary in the setting of a
suspected myeloma, especially if symptomatic. A serum protein
electrophoresis (SPEP) and bone survey should be done first however.
A bone scan (choice B) has low sensitivity for myeloma lesions and has no
role in its workup. Do not confuse a bone survey, which is a series of x-rays
evaluating all of the bones, with a bone scan which is a nuclear medicine
scan.
As stated above, a bone marrow biopsy (choice D) is sometimes necessary
in the setting of a suspected myeloma, especially if symptomatic. However,
a serum protein electrophoresis (SPEP) and a bone survey should be
performed first and this can be done as an outpatient.
As stated above, a bone scan (choice E) has a low sensitivity for myeloma
lesions and has no role in its workup. Do not confuse a bone survey which is
a series of x-rays evaluating all of the bones with a bone scan which is a
nuclear medicine scan.
A 38-year-old man comes to the emergency department complaining of chest
pain. He describes the pain as midsternal and radiating to his back between
his shoulder blades. The pain is not exertional and is associated with some
dyspnea. He has no known significant past medical history other than vague
“problems” with his eyes. He is on a daily multivitamin and has no known
drug allergies. His temperature is 37 C (98.6 F), blood pressure measured at
his right arm is 100/60 mm Hg, pulse is 108/min and regular, and respiratory
42
rate is 20/min. On examination you notice a tall male with long limbs and
fingers. His vascular examination is notable for diminished pulses in his left
arm. His left carotid pulse is also diminished. The pulses over his lower
extremities are brisk and symmetric. Heart and lung examination is
unremarkable. His abdomen is benign. His distal and proximal
interphalangeal joints appear hyperextensible. A chest radiograph shows a
widened mediastinum. An electrocardiogram shows sinus tachycardia at 108
beats per minute with some nonspecific ST, T wave abnormalities. Based
upon the history and physical examination, the most likely diagnosis for the
patient's presenting complaint is
A. cardiac tamponade
B. dissecting aortic aneurysm
C. gastritis
D. myocardial infarction
E. pneumothorax
Explanation:
The correct answer is B. The characteristics of the pain, a midsternal pain
radiating to the back, is a classic description of the chest pain associated
with a dissecting thoracic aneurysm. The presence of asymmetric pulses is
further suggestive of dissection, while the widened mediastinum on chest
radiograph is consistent with the presence of a thoracic aortic aneurysm and
dissection.
Cardiac tamponade (choice A) is unlikely given that there is no mention of
diminished cardiac sounds, or elevated jugular venous pulsations on
physical exam, or evidence of pulsus alternans (beat to beat variability in the
QRS amplitude) on EKG.
Gastritis (choice C) is unlikely given that the patient has no prior history of
gastritis. A diagnosis of gastritis could also would not explain the
constellation of findings on physical exam and chest radiograph.
A myocardial infarction (choice D) is unlikely given that the patient has no
prior history of coronary artery disease, has no risk factors for coronary
disease, has chest pain that is nonexertional, and has an EKG without overt
evidence of ischemia or infarction.
A pneumothorax (choice E) is unlikely given that the patient has no reason
to have this pathology (e.g., a history of antecedent trauma, a history of
emphysema), and has a chest radiograph that is unremarkable for a
pneumothorax.
A 17-year-old girl comes to the office for a follow-up visit after being
diagnosed with iron deficiency anemia. She has been patient of yours since
birth and has always been very healthy. At the last visit, which was 6 weeks
43
ago, she complained of fatigue and she had pale skin and mucus
membranes. She is sexually active with one partner and they use condoms
for contraception. Initial laboratory studies showed:
Hemoglobin 9.5 g/dL
Hematocrit 30%
Ferritin
5 ng/ml
You advised her to begin taking ferrous sulfate and to eat a well balanced,
iron-containing diet with meat. The results of her laboratory studies today
show that she is responding to therapy. The most appropriate next step is to
A. advise her to consider oral contraceptive pills to decrease her
menstrual blood loss
B. check the mean corpuscular hemoglobin (MCV) and red cell
distribution width (RDW)
C. discontinue the ferrous sulfate, but tell her to continue to eat a wellbalanced, iron containing diet
D. do a colonoscopy to check for a source of blood loss
E. make no changes in her current treatment plan
Explanation:
The correct answer is E. This patient most likely has iron deficiency anemia
because she had a low hemoglobin and hematocrit that is responding to
ferrous sulfate therapy and an iron-containing diet. A poor diet and heavy
menstrual bleeding are the most likely cause of iron deficiency anemia in a
woman of childbearing age. Therapy with ferrous sulfate should be
continued for 2-3 more months.
The best next step is to have her continue her current treatment plan, not to
advise her to consider oral contraceptive pills to decrease her menstrual
blood loss (choice A). The history does not tell you that she has very heavy
menstrual bleeding, and even though she has sexual intercourse with one
partner, you would first need to discuss this a lot more before advising her to
use OCPs. The treatment for iron deficiency anemia is ferrous sulfate and
increased dietary intake of iron, not OCPs. OCPs may prevent anemia, but
they do not treat it.
The Centers for Disease Control and Prevention recommend that you check
the mean corpuscular hemoglobin (MCV) and red cell distribution width
(RDW) (choice B) if the anemia does not respond to ferrous sulfate and you
are sure that the patient is compliant. Since she is responding to therapy,
this is not necessary at this time.
It is incorrect for her to discontinue the ferrous sulfate but, telling her to
continue to eat a well-balanced, iron containing diet (choice C) because
even though she is responding to therapy, it is recommended that she
continue for 2-3 more months on ferrous sulfate to replenish iron stores.
A colonoscopy to check for a source of blood loss (choice D) is not
44
necessary at this time in this patient with iron deficiency anemia that is
responding to therapy. The most likely cause of iron deficiency in this patient
is heavy menstrual bleeding and possibly a poor diet. If this patient was a
postmenopausal woman or a man with iron deficiency anemia, you must
think of a gastrointestinal bleed as the cause of anemia. Fecal occult blood
testing and a colonoscopy should be considered for these other patients.
A 42-year-old woman comes into the clinic complaining of intermittent easy
bruising around her eyes and chest, especially after surfing and boogie
boarding with her son. She recently suffered a syncopal episode and a work
up, which included a stress echocardiogram, revealed a hypertrophic heart
with a speckled pattern. When she was discharged from the hospital, she
was given a diagnosis of congestive heart failure with a restrictive pattern. An
endomyocardial biopsy is scheduled for next week. She denies any
significant family history and has been healthy with the exception of easy
bruising, occasionally with vomiting. Routine urinalysis from her recent
admission revealed proteinuria. Upon further questioning, she has suffered
carpal tunnel syndrome bilaterally and occasional numbness and tingling of
her toes. The most useful study to diagnose this patient's condition is
A. complete blood count
B. liver function test
C. morning cortisol level
D. protein immunoelectrophoresis
E. thyroid function tests
Explanation:
The correct answer is D. Protein immunoelectrophoresis is the correct
answer, because this patient has primary systemic amyloidosis which is an
uncommon disease characterized by the extracellular deposition of amyloid
fibrils in multiple organs. Cutaneous manifestations of purpura and
ecchymoses in the periorbital and facial areas are characteristic features in
16% of AL amyloidosus patients. Involvement of the heart, kidney, liver,
thyroid, adrenal, or bone marrow can result in organ dysfunction and early
death. Part of the disease process is clonal proliferation of plasma cells that
produce homogeneous (monoclonal) immunoglobulin proteins (M protein).
Diagnosis of M protein is accomplished with several methods including
serum protein electrophoresis, immunoelectrophoresis, immunofixation, and
capillary electrophoresis.
Although complete blood count (choice A), liver function tests (choice B),
morning cortisol levels (choice C), and thyroid function tests (choice E) may
all useful in the evaluation of a patient with systemic amyloidosis. The most
useful test at this time is immunoelectrophoresis to detect the M protein.
45
You are seeing a 23-year-old woman with diabetes for a routine office visit.
Her regular medications are glyburide and an oral contraceptive pill (OCP).
She is an active smoker and drinks about 4 shots of vodka each weekend.
She eats "lots of meat and potatoes" and exercises 2 times a week. During
your conversation, she mentions that her sister was recently admitted to the
hospital with a "blood clot in her lung." On closer questioning, you also find
that her mother and maternal aunt have been previously admitted for
thromboembolic phenomena. She also mentions that they have been
diagnosed with a problem with their "factor V something." Based on this
information, the most important behavioral modification that you can
recommend to reduce this patient's thromboembolic risk is to
A. follow a low-fat diet
B. increase aerobic exercise
C. maintain tight blood sugar control
D. quit smoking
E. reduce alcohol intake
Explanation:
The correct answer is D. The risk of thromboembolic complications in
women taking oral contraceptive agents who also smoke, especially in the
setting of a possible factor V leiden mutation (to which the patient is hinting),
has been estimated to be as high as 30 times the baseline risk. Therefore,
given the available history, the patient should be strongly counseled to give
up smoking.
Following a low-fat diet (choice A) will help decrease her risk for coronary
and vascular disease in the setting of diabetes, but will do little to decrease
the risk for thromboses.
Increasing aerobic exercise (choice B) will help the patient with her glucose
control and decrease her risk of coronary and vascular disease in the long
run, but do little to decrease her long-term risk for thromboses.
Tighter blood sugar control (choice C) will reduce the patients risk for
complications from diabetes such as large/small vessel disease and renal
disease, but not affect the patient's risk for developing thromboses.
Reducing alcohol use (choice E), while providing a diminished risk for
alcohol related complications, will not affect the patient's risk for developing
thromboses.
A 75-year-old woman comes to the office complaining of a 2-day history of
palpitations. This morning her palpitations were accompanied by some
lightheadedness and nausea. You have been treating her for mitral stenosis
and hypertension. The patient has no history of coronary artery disease or
arrhythmias, and her exercise stress test from 1 year ago was negative. On
46
physical examination, her pulse is irregular ranging from 110 to 140/min and
her blood pressure is slightly lower than usual at 95/70 mm Hg. A middiastolic murmur is audible at the cardiac apex, and her jugular venous
pressure is estimated to be 8 cm H2O. An electrocardiogram demonstrates
atrial fibrillation with rapid ventricular response. You admit the patient to the
hospital and she is given a 10 mg bolus of intravenous metoprolol and her
heart slows to 90/min. Another electrocardiogram still demonstrates atrial
fibrillation and her blood pressure is now 135/85 mm Hg. A heparin infusion is
started. She is observed overnight and ruled out for myocardial infarction.
After discussing treatment options the patient opts to have elective
cardioversion of her atrial fibrillation. Before she can undergo this procedure,
she
A. must have a coronary angiogram
B. must have a negative stress test
C. must have a transesophageal echocardiogram
D. needs digoxin loading for rate control
E. requires anticoagulation for 3 weeks
Explanation:
The correct answer is E. This patient is an elderly female who presented
with 2 days of atrial fibrillation and associated symptoms of lightheadedness
and nausea. The etiology of the atrial fibrillation is stenosis of her mitral
valve. Mitral stenosis with elevated left atrial pressures results in left atrial
enlargement, a predisposing factor for the development of atrial fibrillation.
Atrial fibrillation, in the context of mitral stenosis, may compromise proper
left ventricular filling due to the lack of an atrial contraction. This can result in
a decreased cardiac output with presenting symptoms of dizziness,
lightheadedness, and even syncope. A rapid ventricular response to atrial
fibrillation can decrease ventricular filling time as well, decreasing cardiac
output. Rate control of atrial fibrillation can be accomplished by betablockers, Ca+2 channel blockers, or digoxin. Patients in atrial fibrillation for
>48 hours are at risk for developing atrial thrombi that can embolize.
Anticoagulation initially with heparin is required. Conversion of atrial
fibrillation to sinus rhythm can be accomplished by either electrical or
pharmacological methods. In either case, anticoagulation with warfarin is
essential before elective cardioversion can proceed. In emergency cases, if
the patient is unstable despite rate control, cardioversion can be done after
assessment of the left atrium for mural thrombus by transesophageal
echocardiogram.
The presence of coronary artery disease is not a contraindication for
cardioversion, and the suspicion of ischemic heart disease is low. A
coronary angiogram (choice A) is not required.
Again, the presence of underlying ischemia or coronary artery disease is not
a contraindication for cardioversion. A stress test (choice B) is not required
47
before elective cardioversion.
A transesophageal echocardiogram (choice C) is used to assess the left
atrium for mural thrombus before emergency cardioversion in unstable
patients. This patient is stable and undergoing an elective procedure.
The patient's heart rate appears to be well controlled with beta-blockers and
she does not need digoxin therapy (choice D).
A 62-year-old man comes to the emergency department with severe chest
pain radiating to his back. He says the pain started suddenly 2 hours ago and
is most severe between his shoulder blades. He has no significant past
medical history and does not take any medications. Vital signs are:
temperature 37.0 C (98.6 F) and blood pressure 160/100 mm Hg. Radial
pulses are absent bilaterally. An electrocardiogram demonstrates sinus
tachycardia with a rate of 100/min without evidence for ischemia. A chest xray is normal. A CT scan of the chest is performed and one of the images is
shown. Other images show that the ascending aorta is normal.
The most appropriate initial management for this patient is to
A. administer intravenous beta-blockers
B. administer an intravenous thrombolytic
C. obtain a surgery consult for emergent surgical repair
D. order an MRI and MRA of the chest
48
E. order transesophageal echocardiography
Explanation:
The correct answer is A. The patient is presenting with classic signs and
symptoms of an aortic dissection. The CT of the chest confirms the
diagnosis by demonstrating an intimal flap in the descending aorta. This is a
type B aortic dissection by the Stanford classification system and requires
medical treatment. The initial management goals include elimination of pain
and reduction of systolic blood pressure to 100-120 mm Hg. If systolic
hypertension or pain is present, beta-blockers are used to reduce arterial
pressures.
Administration of an intravenous thrombolytic (choice B) is not indicated in
this patient. Thrombolytics are used in the setting of acute myocardial
infarctions, not in aortic dissections.
Emergent surgical repair (choice C) is not indicated in type B aortic
dissections. Type A dissections which involve any part of the ascending
aorta do require emergent surgical consult and urgent surgical intervention.
The area of the aorta with the intimal tear is usually resected and replaced
with a Dacron graft.
MRI and MRA of the chest (choice D) are not the most appropriate next
steps. The dissection of the descending aorta was already demonstrated on
the CT of the chest and MR imaging is not necessary. If the patient had
renal dysfunction and could not tolerate intravenous contrast, MR imaging
could have been used instead of a CT. An MRI, however, takes more time to
acquire images than a CT and is not indicated in hemodynamically unstable
patients.
Transesophageal echocardiography or TEE (choice E) is not indicated in
this patient. The diagnosis of type B aortic dissection was confirmed by a CT
of the chest. Advantages of TEE include its quick and easy bedside use in
the emergency department, which makes it ideal for patients in an unstable
condition. TEE also can detect involvement of the coronary arteries, aortic
insufficiency, and cardiac tamponade.
A 51-year-old man comes to the office complaining that over the past few
months he has become increasingly lethargic and fatigued and over the past
few days his wife has commented to him that he is “somewhat pale”. He
reports increased shortness of breath with minimal activity. He denies chest
pain, orthopnea, or paroxysmal dyspnea. He also notes that he has had an
overwhelming desire to eat the clay that he uses in his potting and is
concerned that he may have contracted some serious infectious disease. His
past medical history is otherwise unremarkable. He is somewhat pale but in
no distress. His temperature is 37.0 C (98.6 F), blood pressure is 125/80 mm
Hg, pulse is 90/min, and respirations are 14/min. His physical examination is
unremarkable. His stool is heme-positive on digital rectal examination. The
most appropriate next step is to
49
A. determine hematocrit and hemoglobin levels
B. order a reticulocyte count
C. schedule an emergent colonoscopy
D. schedule an emergent sigmoidoscopy
E. schedule an upper endoscopy
Explanation:
The correct answer is A. This patient has rectal bleeding as evidenced by
digital rectal examination (DRE). He also has signs and symptoms of
anemia. It stands that he likely has an anal, sigmoid, or colonic lesion that is
bleeding. However, he has not yet been diagnosed with anemia and in fact
may not have anemia but cancer instead, for example. He has all the classic
signs and symptoms of iron-deficiency anemia including pica (clay eating),
but he has not yet been diagnosed with anemia. For this reason, he must
have a hematocrit and hemoglobin sent. The predicted results are a low
hemoglobin and low hematocrit .
The reticulocyte count is part of the next step in the evaluation of established
anemia, (choice B). A low reticulocyte count suggests marrow failure, while
a high reticulocyte count suggests an appropriate response to the anemia.
A colonoscopy (choice C) or even a sigmoidoscopy (choice D) are both
appropriate diagnostic tests but are not emergent. The likely source of his
bleeding would be colonic and both of these (colonoscopy is superior) are
methods of detecting colonic lesions. He can be referred for theses after
laboratory studies are sent.
There is no reason to suspect that this patient has an upper GI bleed if he
were found to be anemic. Upper endoscopy (choice E) is therefore not
appropriate unless lower endoscopy were to prove negative.
A 62-year-old man comes to the office for a follow up after having an
echocardiogram performed to evaluate a heart murmur. The patient, prior to
the detection of the murmur, had been otherwise well with only mild
hypertension and diet-controlled diabetes. On a routine annual physical
examination, a 4/6 holosystolic murmur was detected. A review of systems
was unremarkable and the patient was referred for an echocardiogram. In
discussion of the results of the echocardiogram, the finding that would
indicate the need for referral to a cardiac surgeon is
A. dilated left atrium
B. ejection fraction of less than 70%
C. end-diastolic dimension greater than 55mm
50
D. end-diastolic dimension of greater than 75mm
E. end-systolic dimension of greater than 55mm
Explanation:
The correct answer is E. This patient has a murmur of mitral regurgitation
(MR). The murmur is classically holosystolic with radiation to the axilla.
Whether the patient is symptomatic or symptom free, an echocardiography
is required to evaluate the presence or absence of actual regurgitation, and
if present, the severity and cause of the MR. A transthoracic echo is
adequate for resolution of the mitral valve and for the detection of an MR jet.
The criteria for referral to a cardiac surgeon are based upon
echocardiographic findings. Specifically, an end-systolic left ventricular
dimension of greater than 45-55 mm or a left ventricular ejection fraction of
less than 55% are reasons for referral and evaluation for valve repair or
replacement.
A dilated left atrium (choice A), an ejection fraction of less than 70% (choice
B), an end-diastolic dimension greater than 55mm (choice C), and an enddiastolic dimension of greater than 75 mm (choice D) are not indications for
a referral to a cardiac surgeon.
A 55-year-old man is in the intensive care unit for atrial fibrillation with rapid
ventricular response. The patient was admitted from the emergency
department after arrival with a blood pressure of 70/50 mm Hg with a heart
rate of 180/min and irregular. After electrical cardioversion, the patient was
started on digoxin and a beta blocking agent and admitted to the ICU.
Frequent checks of the patient by the medical team throughout the night
continue to show that the patient is in atrial fibrillation with a blood pressure
averaging 115/60 mm Hg and a heart rate of 70-90/min. You are called to the
patient's room by one of the floor nurses because the patient is complaining
of chest pain. On arrival, the patient appears well but is sitting upright in bed,
leaning forward, with his hand over his chest. His blood pressure remains at
110/50 mm Hg and his heart rate is irregular at 88/min. Auscultation of the
chest reveals a rub over the left thorax with clear lungs. No murmurs or
gallops are appreciated. While evaluating the patient, he becomes
unresponsive. His radial pulse is barely palpable but is irregular at 110140/min. Two other physicians are performing CPR and the patient has a
stable and good airway. His jugular venous pulsations are visible at the angle
of the mandible and his heart sounds are barely audible with a prominent rub.
Lungs are clear with positive pressure mask breaths. The most appropriate
next step is to
A. attempt needle pericardiocentesis
B. attempt needle thoracocentesis
C. infusion of isotonic saline for blood pressure support
51
D. synchronized cardioversion at 100 Joules
E. unsynchronized cardioversion at 200 Joules
Explanation:
The correct answer is A. Once the diagnosis of pericarditis is made, the
possibility that this patient is having pulseless electrical activity from
pericardial tamponade is high on the differential list. The correct emergent
therapy is to perform needle pericardial drainage at the bedside with the
assistance of the V5 lead of the EKG.
Attempting needle thoracocentesis (choice B) is not likely to be beneficial
since there is no evidence that this patient has a tension pneumothorax.
Again, this can cause PEA, but the therapy must be directed towards the
most likely culprit first. Given that this patient has the classic Beck triad of
tamponade, there is no reason to believe that his hypotension is due to his
AF.
Since hypovolemia is also in the differential of PEA, infusion of isotonic
saline for blood pressure support (choice C) is also reasonable. However,
since the patient has pericarditis and now has an elevated JVP,
hypotension, and diminished heart sounds, this option must be secondary to
ruling out tamponade.
Given this, synchronized cardioversion at 100 Joules (choice D) or
unsynchronized cardioversion at 200 Joules (choice E) will be of no use in
this situation.
A 76-year-old woman is brought to the hospital by her son because of "rapid
breathing." She has advanced Alzheimer disease and is unable to give a
coherent history. She was recently diagnosed with breast cancer. She lives
alone, but normally has a health care aide during the day. The aide was not
available when the son tried to reach her to ask if anything happened. The
son has not seen his mother in 2 months. An accentuated fall in systolic
blood pressure during inspiration would most likely suggest
A. anxiety
B. cardiac tamponade
C. myocardial infarction
D. senile aortic stenosis
E. sepsis
Explanation:
The correct answer is B. This question describes pulsus paradoxus, which is
when there is an accentuated fall in systolic blood pressure (>10 mm Hg)
during inspiration. There is usually a small decrease in blood pressure
52
during inspiration, but it is exaggerated in cardiac tamponade because the
external compression caused by fluid accumulation around the heart leads
to impaired ventricular filling, reduced left ventricular stroke volume, and a
reduction in systolic blood pressure. Cardiac tamponade can occur acutely
after trauma or develop chronically from the accumulation of pericardial fluid
malignancies (she has breast cancer), uremia, infections, collagen vascular
diseases, or radiation.
All of the other choices, anxiety (choice A), myocardial infarction (choice C),
senile aortic stenosis (choice D), and sepsis (choice E), may be associated
with tachypnea, but are not usually direct causes of pulsus paradoxus.
A 78-year-old woman was admitted to the hospital with a new headache,
visual changes, and jaw claudication. The nurse taking care of the patient
calls you to say that no admission orders were written for this patient. You
rush to the floor and review the chart and see that the laboratory studies that
were ordered on admission, have returned and show an erythrocyte
sedimentation rate of 97 mm/hr. You see that a temporal artery biopsy was
done earlier today to rule out giant cell arteritis but nothing else has been
ordered. Her symptoms are unchanged from admission. The next most
appropriate action is to
A. closely observe until the biopsy results are back
B. initiate plasmapheresis
C. start treatment with glatiramer
D. start treatment with intravenous immunoglobulin
E. start treatment with prednisone
Explanation:
The correct answer is E. This patient most likely has temporal arteritis and it
is important that treatment be initiated with prednisone immediately. She
may lose her sight by the time the biopsy results come back.
As stated above, this patient most likely has temporal arteritis and needs
immediate prednisone therapy to prevent blindness. Close observation until
the biopsy results are back (choice A) could have disastrous effects
(permanent blindness).
Intravenous immunoglobulin and plasmapheresis (choices B and D) are not
treatments for temporal arteritis. They are used for other neurological illness
including Guillain-Barre syndrome.
Glatiramer (choice C) is a treatment for multiple sclerosis, not temporal
arteritis.
53
You are seeing a 33-year-old woman for the first time in your office. She is a
recent immigrant from Africa and has not had a primary care physician
previously. She has no medical history, but does complain of fatigue. She is
on no medications and reports no allergies to any medications. She denies
any alcohol, tobacco, or drug use. On review of symptoms she reports heavy
menses regularly. Her blood pressure is 112/67 mm Hg, pulse is 98/min, and
respirations are 21/min. Her nail beds and oral mucosa are slightly pale. The
remainder of the physical examination is unremarkable. Laboratory studies
show that her hematocrit is 28% with microcytosis. Based on the available
information, the most appropriate management of this patient's anemia is to
start her on
A. erythropoietin, intravenously
B. folate, orally
C. iron, orally
D. prednisone, orally
E. thrombopoietin, intravenously
Explanation:
The correct answer is C. The most common cause of a microcytic anemia is
iron deficiency. This is especially true in settings where ongoing iron loss is
present as with heavy menstrual bleeding.
Intravenous erythropoietin (choice A), a stimulator of red cell production, is
used in cases of anemia associated with chronic disease or renal disease. It
does not play a role in the treatment of iron deficiency anemia.
Oral folate (choice B) is incorrect since anemia of folate deficiency is
macrocytic and not microcytic as with this patient.
Prednisone (choice D), an oral steroid, has no role in the management of
iron deficiency anemia.
Intravenous thrombopoietin (choice E), a stimulator of platelet production,
has no role in the treatment of iron deficiency anemia.
54